GMAT Critical Reasoning: Two-Person Arguments
GMAT Critical Reasoning: Two-Person Arguments
com | +91-97395-61394
Also corroborated by
Top-One-Percent is the only institute in the world that makes sure its
(well-prepared) students find nothing unusual / surprising on the test
day.
There’s nothing to panic, though, as there’s nothing new. Just practice the
extra questions given thoroughly.
1. Naima: The proposed new computer system, once we fully implemented it, would operate
more smoothly and efficiently than the current system. So, we should devote the resources
necessary to accomplish the conversion as soon as possible.
Nakai: We should keep the current system for as long as we can. The cost in time and
money of converting to the new system would be greater than any predicted benefits.
2. Raphaela: Forcing people to help others is morally wrong. Therefore, no government has
the right to redistribute resources via taxation. Anyone who wants can help others
voluntarily.
Edward: Governments do have that right, insofar as they give people the freedom to leave
and hence not to live under their authority.
Raphaela and Edward disagree about the truth of which one of the following?
A. Any government that does not permit emigration would be morally wrong to redistribute
resources via taxation.
B. Any government that permits emigration has the right to redistribute resources via taxation.
C. Every government should allow people to help others voluntarily.
D. Any government that redistributes resources via taxation forces people to help others.
E. Any government that forces people to help others should permit emigration.
3. Alexander: The chemical waste dump outside our town should be cleaned up immediately.
Admittedly, it will be very costly to convert that site into woodland, but we have a pressing
obligation to redress the harm we have done to local forests and wildlife.
Teresa: But our town’s first priority is the health of its people. So even if putting the dump
there was environmentally disastrous, we should not spend our resources on correcting it
unless it presents a significant health hazard to people. If it does, then we only need to
remove that hazard.
Which one of the following is the point at issue between Alexander and Teresa?
A. whether the maintenance of a chemical waste dump inflicts significant damage on forests
and wildlife
B. whether it is extremely costly to clean up a chemical waste dump in order to replace it by
a woodland
C. whether the public should be consulted in determining the public health risk posed by a
chemical waste dump
D. whether the town has an obligation to redress damage to local forests and wildlife if that
damage poses no significant health hazard to people
E. whether destroying forests and wildlife in order to establish a chemical waste dump
amounts to an environmental disaster
4. Bart: A mathematical problem that defied solution for hundreds of years has finally yielded
to a supercomputer. The process by which the supercomputer derived the result is so
complex, however, that no one can fully comprehend it. Consequently, the result is
unacceptable.
Anne: In scientific research, if the results of a test can be replicated in other tests, the results
are acceptable even though the way they were derived might not be fully understood.
Therefore, if a mathematical result derived by a supercomputer can be reproduced by other
supercomputers following the same procedure, it is acceptable.
The exchange between Bart and Anne most strongly supports the view that they
disagree as to
A. whether a scientific result that has not been replicated can properly be accepted
B. whether the result that a supercomputer derives for a mathematical problem must be
replicated on another supercomputer before it can be accepted
C. the criterion to be used for accepting a mathematical result derived by a supercomputer
D. the level of complexity of the process to which Bart refers in his statements
E. the relative complexity of mathematical problems as compared to scientific problems
5. Dana: It is wrong to think that the same educational methods should be used with all
children. Many children have been raised in more communal environments than others and
would therefore learn better through group, rather than individual, activities. A child’s
accustomed style of learning should always dictate what method is used.
Pat: No, not always. The flexibility of being able to work either on one’s own or in a group
is invaluable in a world where both skills are in demand.
The conversation lends the most support to the claim that Dana and Pat disagree on
which one of the following?
6. Juan: Unlike the ancient Olympic games on which they are based, the modern Olympics
include professional as well as amateur athletes. But since amateurs rarely have the
financial or material resources available to professionals, it is unlikely that the amateurs
will ever offer a serious challenge to professionals in those Olympic events in which
amateurs compete against professionals. Hence, the presence of professional athletes
violates the spirit of fairness essential to the games.
Michiko: But the idea of the modern Olympics is to showcase the world’s finest athletes,
regardless of their backgrounds or resources. Hence, professionals should be allowed to
compete.
Which one the following most accurately expresses the point at issue between Juan
and Michiko?
A. whether the participation of both amateur and professional athletes is in accord with the
ideals of the modern Olympics
B. whether both amateur and professional athletes competed in the ancient Olympic games
upon which the modern Olympics are based
C. whether the athletes who compete in the modern Olympics are the world’s finest
D. whether any amateur athletes have the financial or material resources that are available to
professional athletes
E. whether governments sponsor professional as well as amateur athletes in the modern
Olympics
7. Opponent of offshore oil drilling: The projected benefits of drilling new oil wells in certain
areas in the outer continental shelf are not worth the risk of environmental disaster. The oil
already being extracted from these areas currently provides only 4 percent of our country’s
daily oil requirement, and the new wells would only add one-half of 1 percent.
Proponent of offshore oil drilling: Don’t be ridiculous! You might just as well argue that
new farms should not be allowed, since no new farm could supply the total food needs of
our country for more than a few minutes.
8. Adam: Marking rod edges with reflecting posts gives drivers a clear view of the edges,
thereby enabling them to drive more safely. Therefore, marking road edges with reflecting
posts will decrease the annual number of road accidents.
Aiesha: You seem to forget that drivers exceed the speed limit more frequently and drive
close to the road edge more frequently on roads that are marked with reflecting posts than
on similar roads without posts, and those are driving behaviors that cause road accidents.
Mark: What’s more, Harper’s ideas have had enough time to be adopted if they really
resulted in superior sound. It took only ten years for the Torres design for guitars to be
almost universally adopted because of the improvement it makes in tonal quality.
Which one of the following most accurately describes the relationship between Jane’s
argument and Mark’s argument?
10.Jorge: You won’t be able to write well about the rock music of the 1960s, since you were
just an infant then. Rock music of the 1960s was created by and for people who were then
in their teens and early twenties.
Ruth: Your reasoning is absurd. There are living writers who write well about ancient
Roman culture, even though those writers are obviously not a part of ancient Roman
culture. Why should my youth alone prevent me from writing well about the music of a
period as recent as the 1960s?
Sue: Nonsense. Usually no one bothers to try to observe comets when they are so far from
the Sun. This flare was observed only because an observatory was tracking Halley’s Comet
very carefully.
12.Ingrid: Rock music has produced no songs as durable as the songs of the 1940s, which
continue to be recorded by numerous performers.
Jerome: True, rock songs are usually recorded only once. If the original recording continues
to be popular, however, that fact can indicate durability, and the best rock songs will prove
to be durable.
Art critic: But in a copy of Veronese’s painting made shortly after Veronese died, the cloak
is red. It is highly unlikely that a copyist would have made so major a change so soon after
Veronese’s death.
The art critic’s response to the curator would provide the strongest support for which
one of the following conclusions?
A. The copy of Veronese’s painting that was made soon after the painter’s death is
indistinguishable from the original.
B. No painting should be restored before the painting is tested with technologically
sophisticated equipment.
C. The proposed restoration will fail to restore Veronese’s painting to the appearance it had at
the end of the artist’s lifetime.
D. The value of an artist’s work is not necessarily compromised when that work is tampered
with by later artists.
E. Veronese did not originally intend the central figure’s cloak to be green.
14.Tony: A new kind of DVD has just been developed. It lasts for only half as many viewings
as the old kind does but costs a third as much. Therefore, video rental stores would find it
significantly more economical to purchase and stock movies recorded on the new kind of
DVD than on the old kind.
Anna: But the DVD itself only accounts for 5 percent of the price a video rental store pays
to buy a copy of a movie on video; most of the price consists of royalties the store pays to
the studio that produced the movie. So, the price that video rental stores pay per copy would
decrease by considerably less than 5 percent, and royalties would have to be paid on
additional copies.
Lin: No, the companies would still have sufficient leverage in negotiations if they hired
temporary replacements.
Which one of the following statements is most strongly supported by the exchange
between David and Lin?
A. David does not believe that the freedom to hire temporary replacements gives companies
any leverage in their negotiations with strikers.
B. David and Lin believe that companies should be allowed as much leverage in negotiations
as the striking employees.
C. David and Lin disagree over the amount of leverage companies lose in their negotiations
with strikers by not being able to hire permanent replacements.
D. David and Lin disagree over how much leverage should be accorded companies in their
negotiations with strikers.
E. Lin believes it is unfair to forbid companies from hiring permanent replacements for their
striking employees.
While it may have initially seemed as if their arguments were related in many ways, there is
actually only one point of intersection, or overlap, between the two: when to convert to the
new system. Both participants support their claims with a supporting premise, but these
premises do not overlap. Naima uses a comparison of system quality to justify her point, while
Nakai cites the cost of making the conversion to justify his point. So in the end, they disagree
over the main conclusion. Thus, the only point of overlap, and therefore the only possible point
of disagreement, is when to convert to the new system. Answer (E) is the correct answer.
Notice that some of the other answer choices are tempting because they seem related, and they
seem like points of disagreement. Take (B), for example. It seems like Naima would believe
that “it is essential to have the best computer system available,” and that Nakai wouldn’t
necessarily agree with this (especially if it’s too expensive). But neither of them ever really
addresses this point directly. Don’t assume!
2. B
Notice that there is just one point of intersection: the main conclusion (governments’ rights to
redistribute resources through taxation). This shouldn’t be surprising. Remember, the overlap
will generally occur on 1) the main conclusion, or 2) a supporting premise. Raphaela argues
that governments do NOT have this right, while Edward argues that they DO have this right.
Each participant uses a supporting premise to support his or her argument, but there is no
overlap between these premises.
Thus, there is just one possible point of disagreement. Let’s look at the answer choices:
(A) Any government that does not permit emigration would be morally wrong to
redistribute resources via taxation.
This does not address the issue of government rights. Eliminate it.
(B) Any government that permits emigration has the right to redistribute resources via taxation.
This addresses a government’s right to redistribute resources (the point of overlap), but it also
mentions emigration, which is a topic unique to Edward’s argument. Let’s put this one on hold
for now.
(C) Every government should allow people to help others voluntarily.
Answer (C) discusses the concept of people helping others (unique to Raphaela’s argument)
and fails to mention anything about the right of governments. Eliminate it.
(D) Any government that redistributes resources via taxation forces people to help others.
Answer (D) mentions forcing people to help others (unique to Raphaela’s argument) and fails
to mention anything about the right of governments. Eliminate it.
(E) Any government that forces people to help others should permit emigration.
This mentions forcing people to help others (unique to Raphaela) and emigration (unique to
Edward) and fails to mention anything about the right of governments. Eliminate it.
Therefore, (B) is the correct answer as it addresses the one point of overlap. It does mention
emigration (unique to Edward’s argument), but this is appropriate. After all, Raphaela argues
that NO government has the right to redistribute resources through taxation. Edward argues
that governments do have this right, as long as they allow emigration. In other words, the two
disagree only if emigration is allowed. Thus, this qualification must be part of the answer.
3. D
Notice that it’s left unclear whether Teresa actually agrees or disagrees about the need to clean
up the waste dump. She only states that if it were to be done, it should be done for particular
reasons and within certain parameters.
So, what is the point of disagreement? Is it Alexander’s second sentence, that it would be very
costly to convert to woodland? No. Teresa doesn’t comment on that.
What about Alexander’s final point, that we have a pressing obligation to redress the harm we
have done to local forests and wildlife? Yes, Teresa does disagree with this, albeit in a
somewhat indirect fashion. She says that the only consideration is whether there is a risk to
people’s health. Let’s evaluate the answers:
(A) is something Alexander would agree with, but Teresa may or may not. We don’t actually
know.
(B) is something Alexander would agree with, while Teresa may or may not.
(C) is not mentioned by either person.
(D) would be quite acceptable to Alexander—his position is that the town has an obligation to
redress the harm done to the environment. Teresa would definitely disagree, so this must be
the right answer.
(E) is interesting in that Alexander definitely thinks harm has been done, but he never mentions
the word “disaster.” Furthermore, Teresa does not reveal her own opinion about this matter,
so we can eliminate the choice for that reason. Her argument simply says “even if” it is a
disaster—we don’t know whether she believes it is.
4. C
Bart says the result is unacceptable because the manner in which it was derived cannot be
comprehended by humans. Anne says that the result is acceptable if it can be reproduced by
other supercomputers, and that it isn’t necessary that humans fully understand it.
(A) is a tempting choice. But do we actually know what Bart’s opinion is about a process being
verified via replication? No. And, while we know Anne believes that replication can make
results acceptable, do we know whether she believes that replication is required for
acceptance? No.
(B) is another tempting choice. Read carefully. Does Anne think that the result must be
replicated in order to be accepted? Not quite. We simply know that Anne believes replication
allows results to be accepted. This is a bit of conditional logic. While Anne says that replication
would be sufficient for this result to be accepted, she does not say that it is necessary.
(C) is better. Bart believes that, since the method used for this result is incomprehensible, the
result is unacceptable. Anne, on the other hand, believes that if this result can be replicated, it
is acceptable. Bart focuses on understanding as his criterion, while Anne focuses on
replication.
(D) is out because Anne does not offer an opinion on the complexity.
(E) is out because science vs. math is not mentioned by either Bart or Anne.
5. B
Dana makes the point that a child’s accustomed style of learning should always dictate the
method of education used, and Pat responds by saying, “No, not always.” Why? Because she
feels that the ability to work in different ways—on one’s own or in a group—is an invaluable
tool in today’s world.
Answer choice (B) is the only one about which we know there are strong feelings on both
sides. We know for sure that Dana disagrees with this statement, because she thinks the child’s
accustomed style should always dictate the educational method. We have a strong sense Pat
agrees with this statement. We know she doesn’t think the child’s accustomed learning style
should always dictate the educational method, and we know she feels this way because she
thinks it’s important for children to learn to work in a variety of ways.
(A) is the correct answer. Do not be fooled by the presence of “amateur.” Juan would say “no”
to professionals on the basis of fairness, Michiko would say “yes” on the basis of having the
finest athletes.
(B) is addressed only by Juan.
(C) is not actually supported by either person. Michiko says the games should showcase the
world’s finest, but does not actually state whether they do.
(D) is not addressed by Michiko, and it goes beyond what is stated by Juan. He does not say
that amateurs never have financial resources equal to pros; he just says they rarely do.
(E) is not addressed by either person.
7. D
Conclusion: The projected benefits of drilling new oil wells in certain areas in the outer
continental shelf are not worth the risk of environmental disaster.
Supporting Premises: The oil already being extracted from these areas currently provides
only 4 percent of our country’s daily oil requirement, and the new wells would only add one-
half of 1 percent.
Now, let’s consider the proponent’s rebuttal relative to the structure of the original argument:
“Don’t be ridiculous!
“You might just as well argue that new farms should not be allowed ….”
“… since no new farm could supply the total food needs of our country for more than a few
minutes.”
Answer choice (D) represents this, and is therefore the correct answer.
Let’s review the other answer choices quickly:
(A) offering evidence in support of drilling that is more decisive than is the evidence offered
by the drilling opponent
The proponent could have done this, but did not. In fact, she did not offer any direct evidence,
one way or the other, that related to drilling.
(B) claiming that the statistics cited as evidence by the drilling opponent are factually
inaccurate
This is incorrect. The proponent is questioning the use of the stats, not their accuracy.
(C) pointing out that the drilling opponent’s argument is a misapplication of a frequently
legitimate way of arguing
The proponent does try to point out flaws in the reasoning structure of the opponent’s
argument, but we have no direct indication that the drilling opponent’s way of arguing is
usually legitimate. Perhaps it’s always illegitimate!
(E) proposing a conclusion that is more strongly supported by the drilling opponent’s evidence
than is the conclusion offered by the drilling opponent The proponent does not present a
conclusion that can be related to the evidence presented by the opponent, and therefore this
answer cannot represent the correct procedure.
8. C
Adam says that marking road edges will allow drivers to drive safely and thus decrease
accidents. Aiesha says that drivers actually drive unsafely on roads with marked edges. How
does this relate to Adam’s argument? It’s evidence that points to the exact opposite conclusion!
With this contradictory aspect in mind, we should go to the choices.
(A) is not quite right. She doesn’t argue with the fact that the marked edges give a clear view.
(C) seems good at first glance—she’s definitely raising a point that challenges Adam’s view.
Does he assume that “facilitating safe driving will result in safer driving”? Sure. He thinks that
if we enable people to drive more safely, there will actually be fewer accidents. This is our
answer.
(D) is kind of like (A). This is not quite right. In fact, Aiesha seems to believe the opposite.
View of the road is relevant; it just increases, rather than decreases, accidents in this case.
(E) is a tempting choice. Read it carefully. Is that the claim that Aiesha is undermining? Not
quite. First of all, she does not even think that the marked edges will, in fact, result in “safer
driving,” and we can eliminate the choice for this reason. Second, if we did not spot that error,
we know that Aiesha’s statements support rather than undermine the idea that something
(marking edges) does not necessarily lead to fewer accidents.
9. E
First take a close look at the statements made by Jane and Mark.
Jane’s position: Jane concludes that Professor Harper’s ideas are valueless because there is no
way to evaluate a guitar sound and determine what constitutes a better-sounding guitar.
Mark’s position: Mark also agrees that Professor Harper’s ideas are valueless, but Mark’s
reasoning is that if Harper’s ideas really worked, then they would have been adopted by now.
In making this analysis, Mark reveals that he believes there is a way to determine that one
guitar sounds better than another.
Mark’s initial comment of “What’s more” leads most people to believe he is in complete
agreement with Jane. Yes, he agrees with her conclusion, but his reason for doing so is
completely contrary to Jane’s reason. Mark actually misinterprets Jane’s claim, and this is why
he says “What’s more,” as if he is adding an additional piece of information that supports her
position. He is not; the premise that he uses contradicts Jane’s premises. If you simply accept
“What’s more” to mean that he is in complete agreement with Jane, you will most certainly
miss the question, and have no idea you have done so.
Answer choice (A): Mark does not address a weakness in Jane’s argument or show how one
could be overcome. Do not mistake the use of “What’s more” to automatically mean that he is
adding something helpful to the situation.
Answer choice (B): Mark’s argument does not have a premise in common with Jane’s
argument; rather, Mark’s argument has the conclusion in common with Jane’s argument.
Answer choice (C): This is a very clever Reverse Answer choice. The answer states: “Mark
and Jane use similar techniques to argue for different conclusions.”
In fact, the following happens in the stimulus: “Mark and Jane use different techniques to
argue for similar conclusions.”
Answer choice (D): An argument is the sum of the premises and conclusion. Although Mark
restates Jane’s conclusion, he does not restate her premises. Therefore, he does not restate her
argument and this answer is incorrect.
Answer choice (E): This is the correct answer. As discussed in the argument analysis, Jane
believes that there is no way to evaluate the merit of a guitar’s sounds. On the opposite side,
Mark’s response indicates he believes that there is a way to evaluate the merit of a guitar’s
sound (“because of the improvement it makes in tonal quality”) and thus the two have
conflicting premises.
10. D
Jorge’s Argument
Premise: Rock music of the 1960s was created by and for people who were then in their teens
and early twenties.
Ruth’s Argument
Premise: There are living writers who write well about ancient Roman culture, even though
those writers are obviously not a part of ancient Roman culture.
Premise: Why should my youth alone prevent me from writing well about the music of a period
as recent as the 1960s?
Note that the question stem asks you to identify how Ruth responded. When two-speaker
stimuli are combined with Method of Reasoning questions, you are typically asked to identify
the reasoning of only one of the speakers (often the second speaker). However, you must still
understand the argument of the other speaker as the answer choices often refer to it.
Answer choice (A): Ruth does not challenge Jorge’s claim about her age. To the contrary, she
seemingly admits he is correct when she says “Why should my youth alone...”
Answer choice (B): Although Ruth uses an example that cites culture, she does not clarify a
definition of popular culture, and certainly not one left implicit in Jorge’s argument.
Answer choice (C): This is a Half Right, Half Wrong answer. The first part of the answer
choice—“using the example of classical culture”—does occur in Ruth’s response, but she does
not use that example “in order to legitimize contemporary culture as an object worthy of
serious consideration.”
Answer choice (D): This is the correct answer. An analogy is a comparison between two
items. In argumentation, analogies are often used to clarify the relationship between the items
or reveal a fundamental truth about one of the items, as in “To better understand the operating
system of your computer, think of it as the brain of your system.” The use of “brain” in the
preceding sentence is the analogy.
Analogies can be used to challenge a position or support a position, but their strength often
rests on the relevant similarities between the two items or scenarios. In the next chapter we
will discuss False Analogies, where an author uses an analogy that is dissimilar enough to be
nonapplicable. As referenced in this answer choice, Ruth analogizes writing about Roman
culture to writing about the 1960s to show that it is not unreasonable that someone who was
an infant can write about that time period. Jorge’s assumption is that if a person was not a teen
or older during the 1960s, then they cannot write well about the music of that period. Since all
elements described in the answer choice occur and the answer describes the method used by
Ruth, this is the correct answer.
Answer choice (E): Ruth does not attack Jorge’s qualification to make his argument, just his
pronouncement that she will not be able to write well about the rock music of the 1960s.
11. D
Anne’s Argument
Premise: Halley’s Comet, now in a part of its orbit relatively far from the Sun, recently flared
brightly enough to be seen by telescope.
Premise: No comet has ever been observed to flare so far from the Sun before.
Conclusion: Such a flare must be highly unusual.
Sue’s Argument
Premise: Usually no one bothers to try to observe comets when they are so far from the Sun.
Premise: This flare was observed only because an observatory was tracking Halley’s Comet
very carefully.
As is often the case with two-speaker stimuli, the speakers disagree. In this case, Anne uses
causal reasoning to indicate that the cause of the sighting is unusual activity with Halley’s
comet:
Sue counters by citing an alternate cause: no one has been looking for such a flare.
NO = no one bothers to try to observe comets when they are so far from the Sun
NCO = no comet has ever been observed to flare so far from the sun
NO caused NCO
The problem now becomes an exercise in figuring out how the test makers will describe the
alternative cause cited by Sue.
Answer choice (A): Sue does not comment on the use of the term “observed” (other than to
explain why the flare was observed).
Answer choice (B): Although Sue cites an explanation that is inconsistent with Anne’s claim,
she does not point out an inconsistency between two of Anne’s claims
Answer choice (C): Remember, evidence is the same as premises. Does Sue contradict Anne’s
premises?
No, she only contradicts her conclusion. Do not be drawn in by the word “nonsense.” That
word is used to attack the conclusion, not the premises of the argument.
Answer choice (D): This is the correct answer. In this answer, the alternate cause is described
as an “alternative explanation.” In most cases, a causal counterargument can be described as
offering an alternative explanation.
Answer choice (E): This is a Reverse Answer. The answer appears as follows: “undermining
some of Anne’s evidence while agreeing with her conclusion”
If the answer choice was reversed in the following manner, it would be correct: “undermining
her [Anne’s] conclusion while agreeing with some of Anne’s evidence”
The evidence she agrees with is the first sentence of Anne’s argument (the premise in the
second sentence is not directly addressed).
12. D
The heart of Ingrid’s argument is that durability is measured by how many times a song is
recorded, and using this standard, rock music songs are not as durable as songs from the 1940s.
Jerome admits that rock music songs are not typically recorded multiple times, but he then
introduces a new way of judging durability—one based on the continuing popularity of the
original recording.
Answer choice (A): Jerome does not misinterpret the claim. He starts off by saying, “True,
rock songs are usually recorded only once,” and this a perfect characterization of part of
Ingrid’s statement.
Answer choice (B): This is a good example of an answer that might be kept as an initial
Contender.
However, as you further consider the answer, you must identify the “contradiction” mentioned
in the answer choice. Does Jerome show that Ingrid’s claim must lead to a contradiction? No.
Note that there is a difference between a speaker contradicting an argument (as Jerome does
here) and a person making a statement that leads to an internal contradiction (known as a self-
contradiction.) An example of a self-contradiction would be:
“Everyone should join our country club. After all, it’s an exclusive group that links many of
the influential members of the community.”
The self-contradiction occurs when the speaker says “Everyone should join” and then follows
that by saying that it is “an exclusive group.” Exclusive, by definition, means that some people
are excluded.
Answer choice (C): As discussed in answer choice (A), Jerome accepts the evidence presented
by Ingrid.
Because he does not undermine the truth of the evidence used by Ingrid, this answer is
incorrect. Again, evidence is another way to say “premise.” We know that Jerome disagrees
with Ingrid’s conclusion, but that does not mean that he disagrees with her premise.
Answer choice (D): This is the correct answer. Jerome’s standard for judging durability is the
popularity of the original. This contrasts with Ingrid’s standard, which is the re-recording of
the song. The point at issue is the definition of durability.
Answer choice (E): Again, use the Fact Test on this answer. Where does Jerome claim that
Ingrid’s knowledge is incomplete? As we discussed previously, he has admitted that her
premise is true.
13. (C)
By asking for a conclusion supported by the art critic’s response, this question essentially asks
you to plant a big “Therefore” in front of each answer choice and choose the one that most
logically follows from the critic’s remarks. In response to the claim that the change from red
to green is justified (which is the curator’s main point), the critic points out counterevidence
suggesting that Veronese’s intended color was in fact red. The upshot, therefore—
as (C) states—is that changing the color from red to green is not likely to restore the painting
to its state during the painter’s life. In the end, the restoration would not be “justified.” (A)’s
sweeping assertion about the Veronese copy does not follow from the critic’s limited evidence
about one particular element of the copy—the cloak.
(B) Testing, whether sophisticated or crude, is outside the scope here. And the critic is
concerned with the merits of one proposed Veronese restoration, not the merits of restoration
in general.
(D) “Tampering”? The critic is talking about a Veronese copy. And that copy has nothing to
do with “the value of” the original Veronese. Furthermore, the very specific topic and scope
are the merits of one proposed change to a Veronese painting. (E) Mentally inserting the word
“Therefore” between the critic’s statement and choice (E) should confirm why the latter is
wrong. There’s a big gap between the evidence about the post-mortem copy and (E)’s
proposed conclusion about Veronese’s “original intentions.”
14. D
Tony asserts that video stores would make out economically if they bought and stocked movies
recorded on the new kind of DVD. The decreased lifespan of the new tapes, Tony reasons, is
outweighed by their cheaper cost. Anna’s reply begins with the Contrast Keyword “But,” so
no doubt she takes exception to Tony’s advice for the video stores. She points out that the cost
of the DVD is only a small fraction of the overall cost of buying a videotaped movie, the bulk
of the expense residing in the royalty fee. Each time a tape wears out (and these new-fangled
ones wear out faster than the older ones), a store incurs another royalty fee. It’s not hard to see
where Anna is going with this: The overall cost savings Tony touts will be very minimal, if
they exist at all, because the area in which Tony’s savings accrue represents only a tiny fraction
of the overall cost. That’s (D).
(A) Anna passes no judgment on the price of royalties. She merely points out the cost
breakdown to show that Tony hasn’t considered the entire picture, not to bemoan “excessive”
royalty charges.
(B) goes too far. Anna speaks mostly to the issue of price to imply that Tony’s suggestion is
not cost effective. That’s far from a paean to high-quality DVDs or a recommendation of what
the stores should do.
(C), (E) The cost to customers is outside the scope. Both speakers are concerned only with the
cost to the video stores.
15. (C)
David thinks that not allowing companies to hire permanent replacements for striking
employees is unfair, since that would place them in a weak bargaining position with the
strikers. Lin claims that companies would maintain leverage if they hire temporary
replacements. It’s not easy to prephrase the correct inference, so let’s evaluate the choices,
keeping a close eye on the scope of each statement.
(A) The scope of David’s argument is limited to the option of hiring permanent replacements.
It’s impossible to know what effect he thinks temporary replacements will have on the
situation.
(B), (D) The speakers argue over what will be the case in the event of a strike, not what should
be the case, so both of these fall outside the scope. True, David finds the situation “unfair,”
but there’s still no way to know from these statements exactly how much leverage these two
think companies should have. So, we can’t say that they believe the leverage should be
equal (B), nor can we infer that they disagree on the relative amount of leverage each group
should have (D).
(C) fits the bill: While David believes companies will have “little” leverage if they can’t hire
permanent replacements, Lin thinks hiring temporary replacements would afford them
“sufficient” leverage in their battle with strikers. So, David and Lin disagree over how much
leverage companies will lose in a strike situation if forbidden to hire permanent replacements.
(E) describes David’s position. If anything, it would be easier to infer the opposite of (E),
considering that Lin believes there’s an alternative to hiring permanent employees that
wouldn’t entail sacrificing bargaining power.
1. Carolyn: The artist Marc Quinn has displayed, behind a 3. Ilana: Carver’s stories are somber and pessimistic,
glass plate, biologically replicated fragments of which is a sure sign of inferior writing. I have
Sir John Sulston’s DNA, calling it a “conceptual never read a single story of his that ends happily.
portrait” of Sulston. But to be a portrait,
Gustav: Carver was one of the finest writers of the past
something must bear a recognizable resemblance
30 years. Granted, his stories are characterized
to its subject.
by somberness and pessimism, but they are also
Arnold: I disagree. Quinn’s conceptual portrait is wryly humorous, compassionate, and beautifully
a maximally realistic portrait, for it holds structured.
actual instructions according to which Sulston
On the basis of their statements, Ilana and Gustav are
was created.
committed to disagreeing over whether
The dialogue provides most support for the claim
(A) Carver’s stories are truly compassionate
that Carolyn and Arnold disagree over whether the
(B) Carver’s stories are pessimistic in their vision
object described by Quinn as a conceptual portrait of
(C) stories that are characterized by somberness and
Sir John Sulston
pessimism can appropriately be called humorous
(A) should be considered to be art (D) stories that are well written can be somber and
(B) should be considered to be Quinn’s work pessimistic
(C) bears a recognizable resemblance to Sulston (E) there are some characteristics of a story that are
(D) contains instructions according to which Sulston decisive in determining its aesthetic value
was created
(E) is actually a portrait of Sulston 4. Megan: People pursue wealth beyond what their basic
needs require only if they see it as a way of
2. Antonio: One can live a life of moderation by never achieving high status or prestige.
deviating from the middle course. But then
Channen: Not everybody thinks that way. After all,
one loses the joy of spontaneity and misses
money is the universal medium of exchange. So,
the opportunities that come to those who are
if you have enough of it, you can exchange it for
occasionally willing to take great chances, or to
whatever other material goods you may need or
go too far.
want even if you are indifferent to what others
Marla: But one who, in the interests of moderation, think of you.
never risks going too far is actually failing to live
Megan and Channen disagree over whether
a life of moderation: one must be moderate even
in one’s moderation. (A) people ever pursue wealth beyond what is
required for their basic needs
Antonio and Marla disagree over
(B) it is irrational to try to achieve high status or
(A) whether it is desirable for people occasionally to prestige in the eyes of one’s society
take great chances in life (C) the pursuit of monetary wealth is irrational only
(B) what a life of moderation requires of a person when it has no further purpose
(C) whether it is possible for a person to embrace (D) it is rational to maximize one’s ability to purchase
other virtues along with moderation whatever one wants only when the motive for
(D) how often a person ought to deviate from the doing so is something other than the desire
middle course in life for prestige
(E) whether it is desirable for people to be (E) the motive for pursuing wealth beyond what one’s
moderately spontaneous basic needs require is ever anything other than
the desire for prestige or high status
5. Aaron: A prominent judge, criticizing “famous lawyers 7. Hazel: Faster and more accurate order processing
who come before courts ill-prepared to argue would help our business. To increase profits, we
their cases,” recently said, “This sort of cavalier should process orders electronically rather than
attitude offends the court and can do nothing manually, because customers’ orders will then go
but harm to the client’s cause.” I find the judge’s directly to all relevant parties.
remarks irresponsible.
Max: We would lose money if we started processing
Belinda: I find it natural and an admirable display of orders electronically. Most people prefer to
candor. Letting people know of the damage their interact with human beings when placing orders.
negligence causes is responsible behavior. If we switch to electronic order processing, our
business will appear cold and inhuman, and we
The point at issue between Aaron and Belinda is
will attract fewer customers.
whether
Hazel and Max disagree over whether
(A) ill-prepared lawyers damage their clients’ causes
(B) the judge’s criticism of lawyers is irresponsible (A) electronic order processing is faster and more
(C) a lawyer’s being ill-prepared to argue a client’s accurate than is manual order processing
case constitutes negligence (B) faster and more accurate order processing would
(D) famous lawyers have a greater responsibility to be financially beneficial to their business
be well prepared than do lawyers who are not (C) switching to electronic order processing would be
famous financially beneficial to their business
(E) it is to be expected that ill-prepared lawyers (D) their business has an obligation to be as profitable
would offend the court in which they appear as possible
(E) electronic order processing would appear cold
6. Azadeh: The recent increase in the amount of and inhuman to most of their customers
organically produced food indicates that
consumers are taking a greater interest in the 8. Mary: Computers will make more information available
environment. Thus, there is new hope for a to ordinary people than was ever available
healthier planet. before, thus making it easier for them to acquire
knowledge without consulting experts.
Ben: No, Azadeh, if you interviewed people who buy
organic produce, you’d see that they’re actually as Joyce: As more knowledge became available in
selfish as everyone else, since they’re motivated previous centuries, the need for specialists
only by worries about their own health. to synthesize and explain it to nonspecialists
increased. So, computers will probably create a
Azadeh’s and Ben’s statements provide the most support
greater dependency on experts.
for holding that they disagree about whether
The dialogue most strongly supports the claim that Mary
(A) it is likely that a healthy planet can be maintained
and Joyce disagree with each other about whether
if most people continue in their present eating
habits (A) computers will contribute only negligibly to
(B) people can become healthier by increasing their the increasing dissemination of knowledge in
consumption of organic foods society
(C) people ought to be more concerned about the (B) computers will increase the need for ordinary
environment than they currently are people seeking knowledge to turn to experts
(D) the rise in organic food production shows people (C) computers will make more information available
to have a greater concern for the environment to ordinary people
than they had before (D) dependency on computers will increase with the
(E) people can be persuaded to have a greater increase of knowledge
concern for the environment than they now have (E) synthesizing knowledge and explaining it to
ordinary people can be accomplished only by
computer experts
9. Trisha: Today’s family is declining in its ability to carry 11. Yolanda: Gaining access to computers without
out its functions of child-rearing and providing authorization and manipulating the data and
stability for adult life. There must be a return programs they contain is comparable to joyriding
to the traditional values of commitment and in stolen cars; both involve breaking into private
responsibility. property and treating it recklessly. Joyriding,
however, is the more dangerous crime because it
Jerod: We ought to leave what is good enough alone.
physically endangers people, whereas only
Contemporary families may be less stable than
intellectual property is harmed in the case of
traditionally, but most people do not find that to
computer crimes.
be bad. Contemporary criticisms of the family are
overblown and destructive. Arjun: I disagree! For example, unauthorized use of
medical records systems in hospitals could
Trisha and Jerod disagree over whether the institution of
damage data systems on which human lives
the family is
depend, and therefore computer crimes also cause
(A) adequate as it is physical harm to people.
(B) changing over time
An issue in dispute between Yolanda and Arjun is
(C) valued by most people
(D) not going to survive (A) whether joyriding physically endangers human
(E) no longer traditional lives
(B) whether the unauthorized manipulation of
10. Francis: Failure to become properly registered to vote computer data involves damage to private
prevents one-third of the voting-age citizens of property
Lagonia from voting. If local election boards (C) whether damage to physical property is more
made the excessively cumbersome registration criminal than damage to intellectual property
process easier, more people would register and (D) whether the unauthorized use of computers is as
vote. dangerous to people as is joyriding
(E) whether treating private property recklessly is
Sharon: The high number of citizens not registered to
ever a dangerous crime
vote has persisted despite many attempts to make
registering easier. Surveys show that most of
these citizens believe that their votes would not
make a difference. Until that belief is changed,
simplifying the registration process will not
increase the percentage of citizens registering
to vote.
The main issue in dispute between Francis and Sharon is
(A) whether changing the voter registration process
would be cumbersome
(B) why so many citizens do not register to vote
(C) what percentage of those registered to vote
actually vote
(D) whether local election boards have simplified the
registration process
(E) why the public lacks confidence in the effects of
voting
12. Taylor: Researchers at a local university claim that 14. Mark: To convey an understanding of past events, a
61 percent of the information transferred during historian should try to capture what it was like
a conversation is communicated through to experience those events. For instance, a foot
nonverbal signals. But this claim, like all such soldier in the Battle of Waterloo knew through
mathematically precise claims, is suspect, direct experience what the battle was like, and it
because claims of such exactitude could never be is this kind of knowledge that the historian must
established by science. capture.
Sandra: While precision is unobtainable in many Carla: But how do you go about choosing whose
areas of life, it is commonplace in others. Many perspective is the valid one? Is the foot soldier’s
scientific disciplines obtain extremely precise perspective more valid than that of a general?
results, which should not be doubted merely Should it be a French or an English soldier? Your
because of their precision. approach would generate a biased version of
history, and to avoid that, historians must stick
The statements above provide the most support for
to general and objective characterizations of
holding that Sandra would disagree with Taylor about
the past.
which one of the following statements?
Mark’s and Carla’s positions indicate that they disagree
(A) Research might reveal that 61 percent of the
about the truth of which one of the following?
information taken in during a conversation is
communicated through nonverbal signals. (A) The purpose of writing history is to convey an
(B) It is possible to determine whether 61 percent of understanding of past events.
the information taken in during a conversation is (B) The participants in a battle are capable of having
communicated through nonverbal signals. an objective understanding of the ramifications
(C) The study of verbal and nonverbal of the events in which they are participating.
communication is an area where one cannot (C) Historians can succeed in conveying a sense of
expect great precision in one’s research results. the way events in the distant past seemed to
(D) Some sciences can yield mathematically precise someone who lived in a past time.
results that are not inherently suspect. (D) Historians should aim to convey past events from
(E) If inherently suspect claims are usually false, then the perspective of participants in those events.
the majority of claims made by scientists are (E) Historians should use fictional episodes to
false as well. supplement their accounts of past events if
the documented record of those events is
13. Laird: Pure research provides us with new technologies incomplete.
that contribute to saving lives. Even more
worthwhile than this, however, is its role in
expanding our knowledge and providing new,
unexplored ideas.
Kim: Your priorities are mistaken. Saving lives is
what counts most of all. Without pure research,
medicine would not be as advanced as it is.
Laird and Kim disagree on whether pure research
(A) derives its significance in part from its providing
new technologies
(B) expands the boundaries of our knowledge of
medicine
(C) should have the saving of human lives as an
important goal
(D) has its most valuable achievements in medical
applications
(E) has any value apart from its role in providing new
technologies to save lives
15. Dr. Jones: The new technology dubbed “telemedicine” 17. Jessica: The budget surplus should be used only to
will provide sustained improvement in at least increase government payments to those who are
rural patient care since it allows rural physicians unemployed.
to televise medical examinations to specialists Marcie: A better use of the money would be for a
who live at great distances—specialists who will public works project that would create jobs.
thus be able to provide advice the rural patient
would otherwise not receive. On the basis of their statements, Jessica and Marcie are
committed to disagreeing about the truth of which one
Dr. Carabella: Not so. Telemedicine might help rural of the following?
patient care initially. However, small hospitals
will soon realize that they can minimize expenses (A) Using the budget surplus to increase government
by replacing physicians with technicians who payments to those who are unemployed is
can use telemedicine to transmit examinations to putting the money to good use.
large medical centers, resulting in fewer patients (B) The public deserves to have jobs created when
being able to receive traditional, direct medical there are many people who are unemployed.
examination. Eventually, it will be the rare (C) When there is a choice between increasing
individual who ever gets truly personal attention. payments to the unemployed and funding a
Hence, rural as well as urban patient care will suffer. public works project, the latter should usually be
chosen.
Which one of the following is a point at issue between (D) Creating jobs through a public works project will
Dr. Jones and Dr. Carabella? ultimately benefit the public.
(A) whether medical specialists in general offer better (E) There is a better way to use the budget surplus
advice than rural physicians than increasing government payments to those
(B) whether telemedicine technology will be installed who are unemployed.
only in rural hospitals and rural medical centers
(C) whether telemedicine is likely to be widely 18. Powell: Private waste-removal companies spend
adopted in rural areas in future years 60 percent of what public waste-removal
(D) whether the patients who most need the advice companies spend per customer, yet give their
of medical specialists are likely to receive it customers at least as good service. Private
through telemedicine waste-removal companies, therefore, work more
(E) whether the technology of telemedicine will efficiently.
benefit rural patients in the long run Freeman: Your conclusion is unwarranted. Different
customers have different waste-removal needs.
16. Jean: Our navigational equipment sells for $1,100 and Since private companies, unlike their public
dominates the high end of the market, but more counterparts, can select which customers to serve,
units are sold by our competitors in the $700 to they choose to exclude the potential customers
$800 range. We should add a low-cost model, whom they judge to be the most costly to serve.
which would allow us to increase our overall
sales while continuing to dominate the high end. The issue in dispute between Powell and Freeman is the
Tracy: I disagree. Our equipment sells to consumers who (A) accuracy of the figure of 60 percent with regard
associate our company with quality. Moving into to the difference in service costs between private
the low-cost market would put our competitors in and public waste-removal companies
the high-cost market on an equal footing with us, (B) reason private waste-removal companies are
which could hurt our overall sales. able to offer service comparable to that offered
by public ones while spending less money per
Jean’s and Tracy’s statements most strongly suggest customer
that they disagree over which one of the following (C) ability of private versus public waste-removal
propositions? companies to select which customers to serve
(A) There is a greater potential for profits in the low- (D) likelihood of the local authorities’ turning public
cost market than there is in the high-cost market. waste-removal companies into private ones
(B) The proposed cheaper model, if it were made so that the companies can operate with lower
available, would sell to customers who would service costs than they now incur
otherwise be buying the company’s present model. (E) relationship between the needs of a waste-
(C) The company could dominate the low-cost market removal customer and the amount of money it
in the same way it has dominated the high-cost takes to serve that customer
market.
(D) The company would no longer dominate the high-
cost market if it began selling a low-cost model.
(E) Decreased sales of the high-cost model would result
in poor sales for the proposed low-cost model.
19. Sarah: Some schools seek to foster a habit of 21. Wirth: All efforts to identify a gene responsible for
volunteering in their students by requiring them predisposing people to manic-depression have
to perform community service. But since a failed. In fact, nearly all researchers now agree
person who has been forced to do something that there is no “manic-depression gene.”
has not really volunteered and since the habit of Therefore, if these researchers are right, any
volunteering cannot be said to have been fostered claim that some people are genetically
in a person who has not yet volunteered for predisposed to manic-depression is simply false.
anything, there is no way this policy can succeed Chang: I do not dispute your evidence, but I take issue
by itself. with your conclusion. Many of the researchers
Paul: I disagree. Some students forced to perform you refer to have found evidence that a set of
community service have enjoyed it so much several genes is involved and that complex
that they subsequently actually volunteer to do interactions among these genes produce a
something similar. In such cases, the policy predisposition to manic-depression.
can clearly be said to have fostered a habit of The point at issue between Wirth and Chang is whether
volunteering.
(A) efforts to identify a gene or set of several genes
The main point at issue between Sarah and Paul is responsible for predisposing people to manic-
whether depression have all failed
(A) there are any circumstances under which an (B) it is likely that researchers will ever be able to
individual forced to perform a task can correctly find a single gene that predisposes people to
be said to have genuinely volunteered to perform manic-depression
that task (C) nearly all researchers now agree that there is no
(B) being forced to perform community service can manic-depression gene
provide enjoyment to the individual who is (D) current research supports the claim that no one is
forced to perform such service genetically predisposed to manic-depression
(C) being forced to perform community service (E) the efforts made to find a gene that can produce
can by itself encourage a genuine habit of a predisposition to manic-depression were
volunteering in those students who are forced to thorough
perform such service
(D) it is possible for schools to develop policies that
foster the habit of volunteering in their students
(E) students who develop a habit of volunteering
while in school are inclined to perform
community service later in their lives
Which one of the following most accurately expresses Walter and Larissa are logically committed by what they
the point at issue between Efraim and Felicity? say to disagreeing about which one of the following?
(A) whether products and practices described by (A) whether the poor and the rich are part of the same
journalists as posing health risks do in fact pose social fabric
health risks (B) whether the most successful members of a society
(B) whether the people most likely to suffer anxiety are that society’s least tolerant people
in stressful situations are also the ones most (C) whether the disadvantaged members of society
likely to try to avoid health risks suffer from injustice
(C) whether there are any people who ignore (D) whether those who have the most advantages in
journalists’ advice about avoiding health risks a society are morally obligated to correct that
(D) whether people can reduce risks to their health society’s injustices
by heeding the advice of articles in the popular (E) whether the economically privileged members
press of a society are less exposed to certain sorts
(E) whether the sort of anxiety that results from of injustices than are the economically
efforts to avoid health risks can be overcome disadvantaged
24. Claude: Because of the relatively high number of 25. Mario: The field of cognitive science is not a genuinely
middle-aged people in the workforce, there will autonomous discipline since it addresses issues
be fewer opportunities for promotion into upper- also addressed by the disciplines of computer
management positions. Since this will decrease science, linguistics, and psychology. A genuinely
people’s incentive to work hard, economic autonomous discipline has a domain of inquiry all
productivity and the quality of life will diminish. its own.
Thelma: This glut of middle-aged workers will lead Lucy: Nonsense. You’ve always acknowledged
many people to form their own companies. that philosophy is a genuinely autonomous
They will work hard and thus increase economic discipline and that, like most people, you think of
productivity, improving the quality of life even if philosophy as addressing issues also addressed
many of the companies ultimately fail. by the disciplines of linguistics, mathematics, and
psychology. A field of study is a genuinely
On the basis of their statements, Claude and Thelma
autonomous discipline by virtue of its having
are committed to agreeing about which one of the
a unique methodology rather than by virtue of
following?
its addressing issues that no other field of study
(A) The quality of life in a society affects that addresses.
society’s economic productivity.
On the basis of their statements, Mario and Lucy are
(B) The failure of many companies will not
committed to disagreeing about the truth of which one
necessarily have a negative effect on overall
of the following?
economic productivity.
(C) How hard a company’s employees work is a (A) If a field of study that has a unique methodology
function of what they think their chances for lacks a domain of inquiry all its own, it can
promotion are in that company. nonetheless be a genuinely autonomous
(D) The number of middle-aged people in the discipline.
workforce will increase in the coming years. (B) If a field of study is not a genuinely autonomous
(E) Economic productivity will be affected by the discipline, it can still have a unique
number of middle-aged people in the workforce. methodology.
(C) All fields of study that are characterized by
a unique methodology and by a domain of
inquiry all their own are genuinely autonomous
disciplines.
(D) Any field of study that is not a genuinely
autonomous discipline lacks both a unique
domain of inquiry and a unique methodology.
(E) Any field of study that is not a genuinely
autonomous discipline addresses issues also
addressed by disciplines that are genuinely
autonomous.
26. Anthropologist: After mapping the complete dominance 27. Plant Manager: We could greatly reduce the amount of
hierarchy for a troupe of vervet monkeys sulfur dioxide our copper-smelting plant releases
by examining their pairwise interaction, we into the atmosphere by using a new process. The
successfully predicted more complex forms new process requires replacing our open furnaces
of their group behavior by assuming that with closed ones and moving the copper from
each monkey had knowledge of the complete one furnace to the next in solid, not molten, form.
hierarchy. Since our prediction was so accurate, However, not only is the new equipment
it follows that the assumption we used to reach it expensive to buy and install, but the new process
was in fact true. also costs more to run than the current process,
because the copper must be reheated after it has
Primatologist: Although I agree that your assumption
cooled. So overall, adopting the new process will
helped you make those predictions, your
cost much but bring the company no profit.
conclusion does not follow. You might as well
argue that since we can predict the output of Supervisor: I agree with your overall conclusion, but
some bank cash machines by assuming that these disagree about one point you make, since the latest
machines actually want to satisfy the customers’ closed furnaces are extremely fuel-efficient.
requests, these cash machines must really
The point about which the supervisor expresses
have desires.
disagreement with the plant manager is
Which one of the following is a point about which the
(A) whether the new copper-smelting process releases
anthropologist and the primatologist are committed to
less sulfur dioxide gas into the atmosphere than
disagreeing?
the current process
(A) whether the anthropologist successfully predicted (B) whether the new copper-smelting process is more
the behavior of individual monkeys by use of the expensive to run than the current process
map of the troupe’s dominance hierarchy (C) whether the new process should be adopted in the
(B) whether the output of a bank cash machine can be copper-smelting plant
accurately predicted on the basis of knowledge (D) whether closed copper-smelting furnaces are
of the requests made to it by customers more fuel-efficient than open furnaces
(C) whether vervet monkeys can have knowledge of (E) whether cooling and reheating the copper will
the complete hierarchy of dominance relations cost more than moving it in molten form
that exists within their own troupe
(D) whether the fact that the anthropologist’s 28. Steven: The allowable blood alcohol level for drivers
assumption led to such successful predictions should be cut in half. With this reduced limit,
provides sufficient grounds for the claim that social drinkers will be deterred from drinking
the vervet monkeys had knowledge of their and driving, resulting in significantly increased
dominance hierarchy highway safety.
(E) whether the behavior exhibited by vervet
Miguel: No, lowering the current allowable blood alcohol
monkeys in experimental situations can be used
level would have little effect on highway safety,
as the basis for a generalization concerning all
because it would not address the most important
vervet monkeys
aspect of the drunken driving problem, which is
the danger to the public posed by heavy drinkers,
who often drive with a blood alcohol level of twice
the current legal limit.
Steven and Miguel’s statements provide the most
support for holding that they would disagree about the
truth of which one of the followings statements?
(A) Social drinkers who drink and drive pose
a substantial threat to the public.
(B) There is a direct correlation between a driver’s blood
alcohol level and the driver’s ability to drive safely.
(C) A driver with a blood alcohol level above the current
legal limit poses a substantial danger to the public.
(D) Some drivers whose blood alcohol level is lower than
the current legal limit pose a danger to the public.
(E) A driver with a blood alcohol level slightly
greater than half the current legal limit poses no
danger to the public.
29. Roxanne: To protect declining elephant herds from 31. Raphaela: Forcing people to help others is morally
poachers seeking to obtain ivory, people wrong. Therefore, no government has the right to
concerned about such endangered species should redistribute resources via taxation. Anyone who
buy no new ivory. The new ivory and old ivory
markets are entirely independent, however, so wants can help others voluntarily.
purchasing antique ivory provides no incentive to Edward: Governments do have that right, insofar as
poachers to obtain more new ivory. Therefore, they give people the freedom to leave and hence
only antique ivory—that which is at least 75 not to live under their authority.
years old—can be bought in good conscience.
Raphaela and Edward disagree about the truth of which
Salvador: Since current demand for antique ivory one of the following?
exceeds the supply, many people who are
unconcerned about endangered species but would (A) Any government that does not permit emigration
prefer to buy antique ivory are buying new ivory would be morally wrong to redistribute
instead. People sharing your concern about resources via taxation.
endangered species, therefore, should refrain (B) Any government that permits emigration has the
from buying any ivory at all—thereby ensuring right to redistribute resources via taxation.
that demand for new ivory will drop. (C) Every government should allow people to help
others voluntarily.
A point on which Roxanne’s and Salvador’s views differ (D) Any government that redistributes resources via
is whether taxation forces people to help others.
(A) there are substances that can serve as satisfactory (E) Any government that forces people to help others
substitutes for ivory in its current uses should permit emigration.
(B) decreased demand for antique ivory would cause
a decrease in demand for new ivory 32. Monica: The sculpture commissioned for our town
(C) people should take steps to avert a threat to the plaza has been scorned by the public ever since
continued existence of elephant herds it went up. But since the people in our town do
(D) a widespread refusal to buy new ivory will have a not know very much about contemporary art,
substantial effect on the survival of elephants the unpopularity of the work says nothing about
(E) people concerned about endangered species its artistic merit and thus gives no reason for
should refuse to buy ivory objects that are less removing it.
than 75 years old Hector: You may be right about what the sculpture’s
popularity means about its artistic merit.
30. Lea: Contemporary art has become big business. However, a work of art that was commissioned
Nowadays art has less to do with self- for a public space ought to benefit the public, and
expression than with making money. The work popular opinion is ultimately the only way of
of contemporary artists is utterly bereft of determining what the public feels is to its benefit.
spontaneity and creativity, as a visit to any art Thus, if public opinion of this sculpture is what
gallery demonstrates. you say, then it certainly ought to be removed.
Susan: I disagree. One can still find spontaneous, Monica’s and Hector’s statements commit them to
innovative new artwork in most of the smaller, disagreeing about which one of the following principles?
independent galleries.
(A) Public opinion of a work of art is an important
Lea’s and Susan’s remarks provide the most support for consideration in determining the work’s artistic
holding that they disagree about whether merit.
(A) large galleries contain creative artwork (B) Works of art commissioned for public spaces
(B) most galleries contain some artwork that lacks ought at least to have sufficient artistic merit to
spontaneity and creativity benefit the public.
(C) contemporary art has become big business (C) The only reason for removing a work of art
(D) some smaller art galleries still exhibit creative commissioned for a public space would be that
new artwork the balance of public opinion is against the
(E) contemporary art, in general, is much less work.
concerned with self-expression than older art is (D) The sculpture cannot benefit the public by
remaining in the town plaza unless the sculpture
has artistic merit.
(E) In determining whether the sculpture should remain
in the town plaza, the artistic merit of the sculpture
should be a central consideration.
1. E
2. B
3. D
4. E
5. B
6. D
7. C
8. B
9. A
10. B
11. D
12. D
13. D
14. D
15. E
16. D
17. E
18. B
19. C
20. E
21. D
22. D
23. E
24. E
25. A
26. D
27. B
28. A
29. B
30. D
31. B
32. E
3. (D)
(A) Only Gustav mentions compassion; Ilana never does, so
we cannot be sure she even holds an opinion on Carver’s
compassionate qualities. Eliminate.
1. (E)
(B) Ilana must believe that Carver’s work is pessimistic, since
The key to every Point at Issue question is that BOTH she says so; and the same point is one that Gustav concedes
speakers must have addressed the issue set forth in the (“Granted.”). This is a point of agreement between them.
correct answer. Eliminate.
You might well be able to predict the answer to this question— (C) , like (A), focuses on a feature mentioned only by
Carolyn says this thing on display isn’t a portrait at all, while Gustav and not by Ilana. This time it’s whether Carver’s
Arnold says it’s “a maximally realistic portrait”. That’s (E). If stories are humorous. Eliminate.
you didn’t see that, the Decision Tree makes quick work of
(D) Ilana has an opinion here: she states unequivocally that
this one.
pessimism in a story is “a sure sign of inferior writing.” Gustav
(A) Carolyn doesn’t mention “art.” Eliminate. (In fact, Arnold has an opinion too, but a diametrically opposed one: in Carver,
doesn’t either, but there’s no reason to get that far.) one finds a “fine” writer whose work contains pessimism. This
(B) Carolyn doesn’t address whether or not it’s Quinn’s work, is the point at issue. (D) is correct. For the record:
either; Eliminate. (E) Neither Ilana nor Gustav mentions “aesthetic value,” so we
(C) Carolyn takes a position on this, so keep it on the first cannot pin down either one’s opinion on this term.
pass. Does Arnold talk about a “recognizable resemblance”?
He does not, so eliminate the choice. 4. (E)
(D) Carolyn doesn’t take a position on this directly—does she Speakers can only disagree on terms that are common to
do so implicitly by pointing out that it’s replicated DNA? That both, so you can reject choices that drag in new terms.
question is a distraction we don’t need to deal with: if she The Decision Tree is, as always, our anchor on Point at Issue
does implicitly address the issue, her conclusion is yes—and questions, but apart from that it’s worth noting that three of
so is Arnold’s. There’s no disagreement. the choices—(B), (C), and (D)—all mention a term, rationality/
irrationality, that never appears in either Megan or Channen’s
2. (B) statements. Since neither speaker opines as to that which is
In Point at Issue questions, stay focused on the rational or irrational thinking, none of those choices can
Decision Tree—it’s very easy to get distracted by answer possibly reflect their point of disagreement!
choices related to one argument or the other if you’re not (A) The Decision Tree helps us right away. Megan offers a
methodical. condition necessary (“only if”) to pursuing wealth beyond
Start with Antonio and run through all the answer choices, their basic needs. That does not commit her to the belief that
eliminating any that he doesn’t take a position on. Then, walk some people do pursue wealth beyond their needs. So, in fact,
through any remaining choices and see if Marla’s taken a we cannot be sure whether Megan even believes what (A) is
stand. In this case (as in many Point at Issue questions), the saying. Eliminate! And we have already eliminated the next
third step, asking whether their opinions on a point disagree, three choices for the reasons described above.
merely confirms that the only answer choice remaining after (E) is left over and must be correct. In fact, Megan explicitly
the first two passes is correct. says “Indeed, the desire for prestige and status is the only
(A) Antonio takes a position on this—he ties taking great reason people pursue wealth beyond their needs.” Meanwhile,
chances with “joy of spontaneity”. Keep it on the first pass. Channen says, “Not everybody thinks that way”; those who
Marla doesn’t state a position on desirability, though, so are “indifferent to what others think” (and who therefore
eliminate this one in round two. care not for prestige or status) can amass more money than
(B) Correct. Antonio says it means “never deviating from the they need, essentially telling us that there are other reasons
middle course”; Marla says “never” taking risks is immoderate for people to pursue wealth beyond their needs. A flat-out
in itself. For the record: disagreement, as required by the question.
(A) Does Aaron express an opinion on whether underprepared Max never says that electronic order processing wouldn’t be
lawyers damage their clients? No. He is critical of the judge faster (A)—he simply says it wouldn’t be a good idea because
who comments on it, but we know nothing more than that. it would turn off customers. (B) is a bit more tempting, but
Eliminate. note that Max doesn’t have a problem with faster and more
accurate processing—he has a problem with electronic order
(B) Does Aaron hold a view about the responsibility of the
processing. (Maybe a different way of implementing faster
judge’s comments? Indeed yes. Does Belinda? She sure does.
and more accurate processing would be great with Max.) This
Do those views differ? Yes, and therefore (B) is correct. He
is a tricky point, but watch out for terminology shifts like the
states verbatim that the remarks are irresponsible, and she
move from “electronic order processing” to “faster and more
demurs: they amount to refreshing candor.
accurate processing” in (B). We’ve already noted than Max
Because the Decision Tree has revealed (B) as correct, we and Hazel both want to increase profits, so we can rule out
need spend little time on the others. Belinda would certainly (D). Hazel never addresses whether customers will see the
agree with (C), but Aaron might or might not. The comparison electronic processing as cold and inhuman, so we don’t know
in (D) between famous and little-known attorneys is whether she would disagree with Max on that point (E).
irrelevant to Aaron and Belinda’s disagreement over the
judge’s remarks. And if anything, Belinda would probably 8. (B)
agree with (E) (because of her reference to “damage”), but
Mary declares that computers will render experts no longer
again, there’s no way to tell what Aaron’s opinion on the
essential to ordinary people, while Joyce predicts “a greater
statement would be.
dependency” on experts. So, to the proposition made by
(B), Mary would say “No”, and Joyce would say “Yes.”
6. (D)
Neither woman brings up “knowledge dissemination”
Use the Decision Tree to cut your way through Point at Issue
(A) or “dependency on computers” (D), so whether both
questions.
would agree or disagree or part company, we have no idea.
(A) Does Azadeh hold an opinion as to the likelihood of (C) is an area of agreement between them: Mary states it and
healthy Earth maintenance? No, that question goes beyond the Joyce seems to agree because she makes the parallel with
scope of her assertion that there’s “new hope” for the planet “previous centuries.” As for (E), Mary certainly thinks that
because people are buying more organic produce. Eliminate. (E) is false—she feels that ordinary people can cope on their
(B) Does she have an opinion as to people’s individual health? own—and Joyce might well agree. After all, Joyce says that in
Hard to say, since her phrase “new hope for a healthier the past, “specialists” were needed to synthesize and explain
planet” is so vague, and “greater consumer interest” is knowledge. One can be a specialist without being an expert,
different from health habits. Eliminate as non-clear-cut. so Joyce might well agree with Mary that (E) isn’t strictly true.
(C) Azadeh does seem to believe that people’s environmental
concern ought to be greater, but Ben doesn’t disagree; his 9. (A)
retort simply concerns the selfish motivations of those buying Many students have trouble with Point-at-Issue questions.
organic produce, not that which they ought to care about. This one is fairly straightforward: Trisha and Jerod disagree
Eliminate. over some aspect of the institution of family. Trisha argues
(D) directly expresses the basis of Azadeh’s claim that the that, since today’s family is losing some of its ability to raise
planet has “new hope,” whereas Ben would say, no: the rise children and provide stability for adult life, we must return
in organic food production is a result of people’s personal to traditional values. Jerod responds that, since most people
self-centeredness, period. So (D) is the point at issue. For the don’t care about that loss of stability, we should leave things
record: alone. What is the point at issue? Paraphrase the positions:
Trisha: the family needs improvement. Jerod: No, the family’s
(E) Neither Azadeh nor Ben brings up people’s persuadability,
okay as is. In the language of (A), the disagreement is over
so while we might suspect that they might disagree with this
whether or not the family is adequate as is. Notice that beyond
statement, we cannot be sure.
this fundamental disagreement, everything else mentioned by
our speakers is largely there to support the wrong answers:
7. (C)
(B) Jerod agrees with Trisha that families are changing, but
Both Hazel and Max seem interested in increasing profits, but
disagrees about the importance of that change.
they disagree on whether electronic order processing is the
(C) is a distortion of the second clause in Jerod’s second conclusion that joyriding is a more dangerous crime than
sentence. What “most people do not find...to be bad” is a computer hacking? Or does he object to her evidence for this
decrease in family stability, not the family institution itself. conclusion, namely that only intellectual property, and not
Meanwhile, Trisha takes no position on what “most people” people, are harmed by computer crimes? The truth is, we don’t
do or do not value; she’s more interested in how they should know. From what follows Arjun’s initial exclamation, one might
behave. get the sense that he disagrees with the latter, based on the
(D) A little apocalyptic this one, eh? You won’t find the concept example he presents of how computer crimes can physically
of survival in either person’s argument, so they can’t be harm people. Now, if (D) read “whether the unauthorized use
disagreeing about this. of computers is dangerous to people,” it would be correct no
matter how we interpret Arjun’s objection. Yolanda doesn’t
(E) is a 180 choice. Trisha wants a return to traditional family
think computer crimes physically endanger people, and
values, which implies that families are no longer traditional.
clearly Arjun does—a point of contention, end of story. But
Jerod acknowledges that contemporary families are not
this issue of “as dangerous to people as is joyriding,” well,
traditional. So (E) is, if anything, a point of agreement, not
that’s something different. Just because Arjun argues that
disagreement.
computer crimes are dangerous to people, we can’t infer
the extent to which he feels this danger compares with the
10. (B)
danger associated with joyriding. There’s only one way for
Where do the two speakers part company? we’re asked. (D), as written, to be logically correct: we have to assume that
Francis announces the topic at hand in his first phrase, and Arjun’s “I disagree!” is in reference to Yolanda’s conclusion
though both he and Sharon cover a lot of ground in their that “joyriding is the more dangerous crime,” and that his
remarks, that topic, choice (B), remains the basic issue: Why disagreement is not directed toward the evidence that follows.
don’t potential voters register? Francis argues that it’s because Then, and only then, can (D) accurately express a point at
the registration process is too difficult, while Sharon retorts issue between the two speakers.
that there’s a deeper-rooted attitudinal reason.
Luckily, even in rare cases of ambiguous text, the method of
(A) distorts the use of “cumbersome,” a word that Francis uses eliminating the choices gives you another way to get the point.
to characterize the current process, not the effort needed to Here, the four wrong choices all follow the usual patterns:
change it. And Sharon doubts whether making a change would
(A) Arjun evidently disagrees with Yolanda’s claim that
solve the problem at all, so she takes no position on (A).
joyriding is a more dangerous crime than computer crime, but
(C) The only “percentage” cited is the 33 1/3% figure he doesn’t contest the basic assertion that joyriding in itself is
mentioned by Francis. That’s not really the percentage (C) dangerous to people.
speaks of. And Sharon doesn’t dispute it in any case.
(B) This issue is not in dispute—if anything, the spirit of both
(D) Sharon certainly seems to believe that the registration arguments strongly suggests that the disputants would agree
process has been simplified, but all we know about Francis’s that computer crime involves at least the potential for damage.
position is that he believes the process is cumbersome.
(C) focuses on an entirely new and previously unmentioned
We can’t infer from this that Francis thinks the process has
comparison: damage to physical property vs. damage to
never been simplified in the past; whether simplification has
intellectual property. The true comparison is between a crime
occurred previously is not the issue here.
that poses danger to people vs. a crime that’s a danger
(E) Sharon simply cites survey results that people think their only to intellectual property. Furthermore, what does “more
votes don’t make a difference. She doesn’t stop to consider criminal” mean? The stimulus deals with determining the more
why that’s so. And we can’t be sure where Francis would dangerous crime, and it’s not clear that this means the same
stand, because he clams up before Sharon raises the public thing as “more criminal.”
confidence issue at all.
(E) Like (B), it’s fair to infer that there would be agreement
on this issue. Each person discusses at least one case in
11. (D)
which treating private property recklessly is an element of a
No joyride this one, thanks to a small and unintentional dangerous crime.
ambiguity introduced by the testmakers. We’re alerted to
the presence of a contested issue first by the question 12. (D)
stem, and then by the emphatic assertion at the beginning
The correct answer to a Point at Issue question must fall
of Arjun’s reply: “I disagree!” But here the testmakers are
within the scope of both speakers’ arguments.
uncharacteristically vague regarding the nature of Arjun’s
objection: when he says “I disagree!”, we’re forced to Taylor is understandably skeptical of the researcher’s claim
ask ourselves, “With what?” Does he object to Yolanda’s that exactly 61 percent of information transferred in a
conversation is communicated through nonverbal signals. (E) Laird’s second sentence explicitly cites a value of pure
Essentially, his argument is that the number is far too precise research “apart from its role in” developing life-saving
to be taken seriously, because such exact claims can never be technologies. For her part, when Kim says that saving
scientifically established. lives counts “most of all,” she is implicitly conceding that
Sandra, on the other hand, is much more credible of scientific other results of pure research exist. Hence (E) is a point of
claims—she notes that many scientific disciplines are agreement between the speakers.
extremely precise, so their precision isn’t a reason to doubt
them. But notice what Sandra doesn’t mention: the claims of 14. (D)
the researchers at the local university. Her argument implies As stated in (D), Mark thinks historians should “convey past
that she’s not ready to dismiss their claim just because it’s events from the perspective of participants in those events,”
precise, but she only speaks about scientific claims in general. while Carla disagrees and feels this would lead to a “biased
The Decision Tree will help us find the right answer, by version of history.”
focusing on the scope of each speaker’s opinion and asking Mark and Carla agree that history should aim to convey
whether they’d disagree on each choice. Let’s use it to run an understanding of past events (A). (B) is too specific in
through the choices: discussing only participants in a battle, while Carla and
Taylor has an opinion on (A)—he doesn’t think it’s possible Mark disagreed about historical method in general, and
for research to make such a precise claim. But Sandra doesn’t discussed the battle of Waterloo only as an example. Carla
mention anything about this specific claim, so we can reject wouldn’t disagree with (C)—she believes that Mark’s method
(A) as outside the scope of her argument. The same analysis of history is a bad idea, but never implies that she thinks it
gets rid of (B) and (C): Sandra has no opinion about the is impossible. (E) introduces a new concept by discussing
precision of research on verbal and nonverbal communication, fictional episodes used to supplement incomplete records.
only on the precision of certain scientific disciplines. This is too loose a paraphrase of Mark’s position, and doesn’t
capture the essence of the disagreement.
(D) Taylor thinks that all mathematically precise claims are
suspect, and Sandra argues that many scientific claims can
15. (E)
be both precise and inherently valid. Both speakers have an
opinion on (D), and those opinions differ: we’ve found the For “point at issue,” carefully compare the two speakers’ topic
correct answer. and scope.
We can quickly dispatch (E) to finish the question off: Sandra This one is pretty straightforward. Dr. Jones only discusses
doesn’t mention “inherently suspect claims,” so she can’t telemedicine in terms of rural patient care; he is optimistic.
have an opinion on them or what they mean for most scientific Dr. Carabella concedes that rural care “initially” might
claims. improve, but “eventually,” in the long run, it will suffer. She
answers no to (E), Jones answers yes.
13. (D) (A) is not even a position Jones would necessarily take—the
Use the Decision Tree to locate, quickly and accurately, the specialists give different advice, not necessarily better—but
point at issue between speakers. since Carabella never mentions specialists we don’t know
her position on them. Likewise, since Jones never mentions
(A) Laird’s first sentence indicates that he agrees that new
non-rural hospitals and care we can’t be sure of his view as
technologies are an important result of pure research. But Kim
to whether telemedicine will catch on outside of rural areas,
doesn’t disagree explicitly—she doesn’t even mention new
(B). (C) is certainly a point of agreement between the two
technologies. Eliminate.
doctors. (D) is something that Dr. Jones would agree with
(B) Laird explicitly states that pure research expands insofar as rural patients are concerned; as for Carabella, she
knowledge boundaries. Kim echoes this goal in her (again) never mentions specialists.
assertion that pure research was a necessary part of medical
advancement. Eliminate. 16. (D)
(C) Laird explicitly calls saving lives “worthwhile,” and Kim Another disagreement to disentangle, this one between
prizes saving lives above all else. So here, too, is a point of Jean and Tracy over business strategy. Jean’s happy with the
agreement. Eliminate. business brought in by the high-end navigational equipment,
(D) Expanded knowledge and novel ideas are, to Laird, “more but frets over the fact that competitors are selling more units
worthwhile” than life-saving technology, so he denies that (D) in a cheaper price range. She suggests adding a low-cost
is true. Kim’s main point, on the other hand, is that nothing model to boost sales. Tracey differs: Moving into the low-cost
trumps life-saving technology. So (D) is the point at issue arena would, according to her, damage the public’s perception
between them. For the record: of the company as a quality high-end manufacturer, which
would jeopardize their success in the high-end market and Marcie, a better use of the money would be to fund a public
hurt sales overall. We’re looking for a proposition about which work projects to create more jobs.
the two would disagree, so the best bet is to evaluate each (A) Jessica agrees with this, and so could Marcie. Marcie only
choice asking yourself: “What would Jean think of this? What says that funding a public works project is a better use of the
would Tracy?” When you find them on opposite sides of the money. Eliminate.
fence, you’ll have your answer.
(B) What the public deserves is not discussed by either Jessica
(A) Both speakers talk about sales, not profits. Profit is what’s or Marcie.
left when you deduct expenses from sales revenues, which
(C) may be tempting, but the scope of the disagreement
is one step removed from this disagreement. But even if
involves what to do with a budget surplus. Their statements do
you equated profit with sales, this one doesn’t fly. Neither
not commit them to make the same choice with money from a
seems to argue that the real action is in the low-cost market.
different source.
Jean merely wishes to supplement sales on this level while
maintaining dominance in the high end, while Tracy argues (D) Jessica could certainly agree with this statement but still
that doing so will put the company’s high-end dominance at argue that increasing unemployment payments is a better use
risk. of the money. That leaves (E), which therefore must be right.
For the record:
(B) Jean certainly doesn’t believe that the new low-cost model
will cannibalize their bread-and-butter model; she simply sees (E) is exactly Marcie’s position, and we know Jessica disagrees
the possibility for some extra cash. And Tracy’s not concerned with it.
about this, either—she’s worried that moving into the cheaper
market will allow competitors to even the score up in the high- 18. (B)
end. We have no evidence that either speaker would believe For all the verbiage spouted by Powell and Freeman, their
the assertion in (B), and thus we can’t say that they would points boil down simply to Powell’s “Private waste-removal
disagree about it. companies...work more efficiently” than public ones, and
(C) Neither Jean nor Tracy makes any predictions about Freeman’s “Your conclusion is unwarranted.” Wherefore
dominating the low-cost market. Jean merely thinks it’s a nice unwarranted? Powell’s plaudits for the private companies are
opportunity for some extra sales, while Tracy sees a potential based on the fact that they spend 40% less than public ones
drawback to the proposal. with no loss in quality. Freeman challenges that reasoning by
pointing out that the private companies, who pick and choose
(D) finally talks about the issue that’s important to both. Jean
their customers, favor precisely those for whom they can keep
flat-out says that the company will “continue to dominate the
costs down. Thus, Freeman’s challenge to Powell’s evidence—
high end.” She would therefore take issue with the statement
Powell’s “reason”—is what (B) pinpoints. (B) is code for “Are
in (D). Tracy, on the other hand, would support this assertion
private companies more cost-effective because they’re better,
wholeheartedly—after all, she’s the one who says that getting
or is it because they preselect their customers in a particular
into the low-cost market “would put our competitors in the
way?”
high-cost market on an equal footing with us.” That certainly
sounds like a prediction of the end of the company’s high-end (A) Freeman doesn’t differ with Powell’s statistic, but does
dominance. (D) is the proposition over which the two speakers take issue—as (B) points out—with the reason for it.
would disagree. (C) is half-right (in its focus on the comparison of the two
(E) Jean doesn’t believe there will be a decrease in high-end types of companies) and half-wrong (in its reference to the
sales. Technically, we could say that she would disagree with pre-selection issue). Remember that Powell speaks first; it’s
all of (E), as she never even considers that high-end sales will Freeman who brings up the private companies’ customer
be affected. But we can’t come close to pinning Tracy down selection. Powell never commented on it, so (C) can’t be the
on this issue. She talks only of the danger of losing high-end basis of the two speakers’ disagreement.
dominance, and never indicates one way or the other what this (D) wildly distorts both speakers’ scope by focusing on the
will do to sales in the cheaper market. Too much ambiguity, never-even-hinted-at proposal of turning public companies
not enough disagreement. private.
(E) This disagreement is about a “relationship,” all right, but
17. (E) not the one that (E) highlights: It’s about the relationship
Stick to the terms used in the arguments. between being a private concern and being able to hold down
Jessica argues that the budget surplus should be used to help costs. Powell thinks the answer is the private companies’
people currently unemployed. Marcie disagrees. According to quality, while Freeman credits their ability to select their
customers.
19. (C) explaining his behavior. The critic believes that his political
We’re after the point at issue between Sarah and Paul. condition (being a tyrant) is the root cause of his cruelty,
Clearly, they disagree on some aspect of student volunteering, and not the insanity that goes along with being a tyrant.
and (C) hits the nail on the head: the point at the center of (E) summarizes this dispute.
the argument is whether or not students who are forced to (A) Neither side addresses the issue of criminal liability.
volunteer can establish a genuine interest in community (B) Both sides agree on this point. The writers have diagnosed
service—Paul says yes, while Sarah says no. Stalin with “morbid suspiciousness,” and the critic believes
(A) can be tricky. Sarah would have an opinion on (A), as she that all tyrants are morbidly suspicious.
believes that no one forced to perform an action can truly be (C) Neither side weighs factors with respect to the issue of
considered a “volunteer.” But for all we know, Paul would if Stalin was cruel and/or paranoid. Rather, they concern
agree with her that there are no circumstances under which themselves with why he did the things he did.
forced volunteering is real volunteering. He doesn’t deny that
(D) The critic believes that being a tyrant leads to certain kinds
the students were forced, and never argues that the forced
of mental instability which can lead to cruelty, but that’s not
volunteering is genuine. His point is one step beyond this: He
the same thing as believing that tyranny in general leads to
says that once students are engaged in the activity, they may
cruelty in general.
like it enough to then genuinely volunteer on their own later
on.
21. (D)
(B) may also be a tempting choice, so be careful here. Sarah
Here we have a combative dialogue; we can tell by the stem
and Paul disagree over the ability of school programs to
that there’s a conflict here—a point at issue—and we’re to
foster a habit of volunteering, not over the ability of volunteer
find it. Well, Chang can’t be more helpful in the first line of his
activities to provide enjoyment. Paul believes that forced
rebuttal when he says “. . . I take issue with your conclusion.”
community service can be enjoyable, and thus lead to genuine
It stands to reason therefore that Wirth’s conclusion must
volunteering in the future, but Sarah never addresses this
contain the point at issue. Wirth concludes that based on
“enjoyment” issue.
the research cited, people are not genetically predisposed to
(D) The issue here isn’t whether it’s possible for schools manic depression (or as she puts it, “any claim that people are
to develop policies that foster the volunteering habit, but genetically predisposed is false”). This conclusion is perfectly
rather whether or not this specific policy—forcing students paraphrased in choice (D), and we barely need to even read
to perform community service—is successful in fostering a the rest of Chang’s argument to choose (D) as the answer. (D)
habit of volunteering. The issue in (D) is thus broader than the contains Wirth’s conclusion, and Chang begins by stating that
scope of the dialogue, and therefore is not the basis for the he “takes issue with” Wirth’s conclusion. What more can we
disagreement. want? Thus (D) contains the point at issue. If you did read the
(E), like (D), also goes beyond the scope. The argument is rest of Chang’s argument, you’d see that indeed he states that
concerned with whether or not certain programs help students research evidence does support the possibility of a genetic
to develop the volunteering habit. What happens later in life to predisposition to manic depression.
those who do is not relevant to this discussion. (A) Chang’s admission “I do not dispute your evidence” allows
us to axe (A): he agrees, evidently, with Wirth’s statement that
20. (E) “all efforts to identify a gene responsible . . . have failed.” But
A “point of disagreement” is just a point at issue, but this doesn’t Chang argue that a set of genes have been discovered
question is different in that there is only one speaker. that predispose people to manic depression? Be careful here;
This makes it a little more difficult to distinguish the two he doesn’t. All he says is that research indicates that several
points of view, but keywords help us keep them straight. genes, through complex interactions, might predispose people
The author starts by describing the position of the writers. to the disease. He doesn’t disagree with the notion that any
They believe that Stalin may have been insane in his later efforts to actually identify the precise gene or set of such
years. Further, these writers used this diagnosis to explain genes have thus far failed.
his infamous misdeeds. If the writers believe that Stalin’s (B) The debate is over an interpretation of the current research.
insanity explains his misdeeds, then they must believe that Neither speaker ventures a guess as to what the future of
Stalin’s psychological state factors into the cause of his manic depression research will bring.
misdeeds. “But” (note the keyword signaling a contrast) the
(C) Like (A), “I do not dispute your evidence” allows us to
critic disagrees, claiming that all tyrants suffer from similar
axe (C): Chang does not wish to dispute that “nearly all
psychological states. This implies that the critic believes
researchers now agree that there is no manic-depression
that even if Stalin’s paranoia and morbid suspiciousness
gene.” This is a game of semantics—while Chang doesn’t
contributed to his cruelty, there is another, more primary factor
dispute this point, he does argue the notion that research may Walter believes that injustices can inflict the well-to-do just
support the existence of a manic-depression group of genes. as easily. So, Walter and Larissa disagree on whether the
(E) We simply don’t know how either speaker feels about the economically privileged members of a society are less exposed
thoroughness of the researchers’ efforts, so (E) can’t be a to certain sorts of injustices than are the economically
point at issue between Wirth and Chang. disadvantaged—answer choice (E).
(A) Social fabric? Hopefully, this new term immediately
22. (D) caught your attention. Although Walter and Larissa do indeed
Sometimes the point at issue is pretty obvious. disagree on how injustice affects the privileged and the
disadvantaged, they never debate whether the rich and poor
Efraim argues that because attempts to follow advice from
are part of the same social fabric.
the popular press about the avoidance of health risks make
people anxious, and anxiety is itself a major health risk, (B) This answer choice distorts several things that are
people should disregard health advice in the popular press. mentioned. Inferably, both Walter and Larissa believe that
Felicity clearly states her disagreement with Efraim (“you are at least some economically privileged people “tolerate,”
wrong”), and provides the example of how articles in the that is, allow, injustice against the disadvantaged. Whether
popular press helped people learn about the risks of smoking, this makes these rich folks the “least tolerant people” in the
which in turn led many people to improve their health by society is ambiguous at best. And even if you did make this
quitting smoking. leap, which is unwarranted, you’d have to infer this judgment
for both Walter and Larissa; that is, it wouldn’t be a source of
(D) sums up the dispute quite nicely, and we can use the
conflict between them, so it still can’t be the point at issue
Decision Tree to confirm it’s the right answer: both Efraim
we seek.
and Felicity have an opinion about whether people can
improve their health by heeding advice given in the popular (C) is a 180-answerchoice. Both Walter and Larissa agree that
press. Efraim is committed to a negative view, while Felicity is the disadvantaged members of society suffer from injustice,
committed to a positive view of the issue. We have identified and they both agree that this is bad. They disagree on why it
the point at issue in their argument. is bad.
(A) Efraim implicitly states that the popular press describes (D) Neither Walter nor Larissa discusses who is responsible
actual health risks when he acknowledges that his advice to for correcting social injustices. They both agree that social
ignore the articles seems “paradoxical.” Felicity provides the injustices are bad, they differ on why injustices are bad, but
example of articles about the health risks posed by smoking, neither mentions what should be done to fix the problem.
acknowledging that at least sometimes, the popular press
describes actual health risks. 24. (E)
(B) Efraim mentions something about the articles causing A variation on a standard question type need not be a
anxiety, but never makes the comparison contained in the difficult question.
answer choice. Felicity never discusses any relationship This question probably came as a surprise; we’re all used to
between anxiety and attempts to reduce health risks. This being asked what two speakers disagree about. In fact, this
cannot be a point at issue. is a rare time that the GMAT asks what two speakers agree
(C) Both Efraim and Felicity could agree that there are some about. But if the GMAT throws you a curveball, don’t panic.
people who ignore journalists’ advice about avoiding health risks. The skills you’ve learned at will still get you through the
question. When the two speakers both have an opinion on
(E) Efraim implies that the only way to avoid anxiety due to
an answer choice, we’ll ask, “Are those opinions the
attempts to avoid health risks is to disregard the articles.
same?”
Felicity never discusses anxiety.
(A) Both Claude and Thelma seem to think that economic
23. (E) productivity affects quality of life, not the other way around.
Eliminate.
Larissa and Walter agree on the conclusion, but disagree on
the evidence supporting that conclusion. Walter believes that (B) Thelma would agree with (B), but Claude never mentions
we should not tolerate injustice because it may affect a well- companies failing. Eliminate.
to-do person tomorrow. Larissa believes that we should not (C) is an integral part of Claude’s argument, but Thelma has no
tolerate injustice because it’s a source of social unrest. Larissa opinion on promotions within a company. Eliminate.
doesn’t merely present different evidence however, she (D) was probably very tempting, but there is a fine distinction
affirmatively disagrees with Walter’s reasoning. Larissa states, here: both Claude and Thelma agree that there will be a
in the first sentence of her response, that the well-to-do can
protect themselves against injustice better than the rest.
relatively high number of middle-aged people in the workforce, (C) Vervet monkeys’ capability to know the complete
but neither one says that the number will increase. Eliminate. dominance hierarchy of their group is not at issue; the fact of
(E) At last—both Claude and Thelma give us their opinions that knowledge, and the anthropologist’s assumption of that
on how the number of middle-aged workers will affect fact, is the real issue.
productivity, so they must agree that it will be affected. This is (E) Generalizing the experiment to the behavior of all vervet
the correct answer. monkeys is outside the scope of both arguments.
1. Xavier: Demand by tourists in Nepal for inexpensive 2. Mark: To convey an understanding of past events, a
thangka paintings has resulted in the proliferation historian should try to capture what it was like
of inferior thangkas containing symbolic to experience those events. For instance, a foot
inaccuracies—a sure sign of a dying art form. soldier in the Battle of Waterloo knew through
Nepal should prohibit sales of thangkas to direct experience what the battle was like, and it
tourists, for such a prohibition will induce artists is this kind of knowledge that the historian must
to create thangkas that meet traditional standards. capture.
Yvette: An art form without dedicated young artists Carla: But how do you go about choosing whose
will decay and die. If tourists were forbidden to perspective is the valid one? Is the foot soldier’s
buy thangkas, young artists would cease making perspective more valid than that of a general?
thangkas and concentrate instead on an art form Should it be a French or an English soldier? Your
tourists can buy. approach would generate a biased version of
history, and to avoid that, historians must stick
Yvette responds to Xavier by
to general and objective characterizations of the
(A) denying the existence of the problem that past.
Xavier’s proposal is designed to ameliorate
Carla does which one of the following in disputing
(B) challenging the integrity of Xavier’s sources of
Mark’s position?
information
(C) arguing that Xavier’s proposal, if implemented, (A) contests Mark’s understanding of historical events
would result in the very consequences it is (B) questions Mark’s presupposition that one person
meant to prevent can understand another’s feelings
(D) using an analogy to draw a conclusion that is (C) argues that the selection involved in carrying out
inconsistent with the conclusion drawn by Mark’s proposal would distort the result
Xavier (D) questions whether Mark accurately describes the
(E) showing that the evidence presented by Xavier kind of historical writing he deplores
has no bearing on the point at issue (E) gives reason to believe that Mark’s
recommendation is motivated by his professional
self-interest
3. Adam: Marking road edges with reflecting posts 5. Joanna: The only way for a company to be successful,
gives drivers a clear view of the edges, thereby after emerging from bankruptcy, is to produce the
enabling them to drive more safely. Therefore, same goods or services that it did before going
marking road edges with reflecting posts will bankrupt. It is futile for such a company to try to
decrease the annual number of road accidents. learn a whole new business.
Aiesha: You seem to forget that drivers exceed the Ruth: Wrong. The Kelton Company was a major mining
speed limit more frequently and drive close to operation that went into bankruptcy. On emerging
the road edge more frequently on roads that are from bankruptcy, Kelton turned its mines into
marked with reflecting posts than on similar roads landfills and is presently a highly successful
without posts, and those are driving behaviors waste-management concern.
that cause road accidents.
Ruth uses which one of the following argumentative
Aiesha responds to Adam’s argument by techniques in countering Joanna’s argument?
(A) questioning Adam’s assertion that reflecting posts (A) She presents a counterexample to a claim.
give drivers a clear view of road edges (B) She offers an alternative explanation for a
(B) presenting a possible alternative method for phenomenon.
decreasing road accidents (C) She supports a claim by offering a developed and
(C) raising a consideration that challenges the relevant analogy.
argument’s assumption that facilitating safe (D) She undermines a claim by showing that it rests
driving will result in safer driving on an ambiguity.
(D) denying that the drivers’ view of the road is (E) She establishes a conclusion by excluding the
relevant to the number of road accidents only plausible alternative to that conclusion.
(E) providing additional evidence to undermine the
claim that safer driving does not necessarily 6. A newspaper article on Britain’s unions argued that their
reduce the number of road accidents strength was declining. The article’s evidence was the
decreasing number and size of strikes, as if the reason
4. Robert: Speed limits on residential streets in for the unions’ existence was to organize strikes. Surely,
Crownsbury are routinely ignored by drivers. in a modern industrial society, the calling of a strike is
People crossing those streets are endangered evidence that the negotiating position of the union was
by speeding drivers, yet the city does not have too weak. Strong unions do not need to call strikes.
enough police officers to patrol every street. They can concentrate their efforts on working with
So,the city should install speed bumps and others in the labor market to achieve common goals,
signs such as profitable and humane working conditions.
warning of their presence on residential streets to
The argument criticizing the newspaper article employs
slow down traffic.
which one of the following strategies?
Sheila: That is a bad idea. People who are driving too
(A) questioning the accuracy of the statistical
fast can easily lose control of their vehicles when
evidence that the newspaper article uses
they hit a speed bump.
(B) detailing historical changes that make the
The relationship of Sheila’s statement to Robert’s newspaper article’s analysis outdated
argument is that Sheila’s statement (C) reinterpreting evidence that the newspaper article
uses as indicating the opposite of what the
(A) raises the objection that the problem with which
newspaper concludes
Robert is concerned may not be as serious as he
(D) arguing that the newspaper article’s conclusion
takes it to be
is motivated by a desire to change the role of
(B) argues that the solution Robert advocates is likely
unions
to have undesirable side effects of its own
(E) pointing to common interests among unions
(C) defends an alternative course of action as more
and management which the newspaper article
desirable than the one advocated by Robert
ignores
(D) concedes that the solution advocated by Robert
would be effective, but insists that the reasons
for this are not those given by Robert
(E) charges that Robert’s proposal would have no net
effect on the problem he describes
7. Proponent: Irradiation of food by gamma rays would 9. Dr. Jones: The new technology dubbed “telemedicine”
keep it from spoiling before it reaches the will provide sustained improvement in at least
consumer in food stores. The process leaves no rural patient care since it allows rural physicians
radiation behind, and vitamin losses are to televise medical examinations to specialists
comparable to those that occur in cooking, so who live at great distances—specialists who will
there is no reason to reject irradiation on the thus be able to provide advice the rural patient
grounds of nutrition or safety. Indeed, it kills would otherwise not receive.
harmful Salmonella bacteria, which in
contaminated poultry have caused serious illness Dr. Carabella: Not so. Telemedicine might help rural
to consumers. patient care initially. However, small hospitals
will soon realize that they can minimize expenses
Opponent: The irradiation process has no effect on the by replacing physicians with technicians who
bacteria that cause botulism, a very serious form can use telemedicine to transmit examinations to
of food poisoning, while those that cause bad large medical centers, resulting in fewer patients
odors that would warn consumers of botulism are being able to receive traditional, direct medical
killed. Moreover, Salmonella and the bacteria that examination. Eventually, it will be the rare
cause botulism can easily be killed in poultry by individual who ever gets truly personal attention.
using a safe chemical dip. Hence, rural as well as urban patient care will
The opponent’s argument proceeds by suffer.
(A) isolating an ambiguity in a crucial term in the Dr. Carabella uses which one of the following strategies
proponent’s argument in responding to Dr. Jones?
(B) showing that claims made in the proponent’s (A) listing a set of considerations to show that a
argument result in a self-contradiction prescribed treatment that seems to be benefiting
(C) establishing that undesirable consequences result a patient in fact harms that patient
from the adoption of either one of two proposed (B) describing the application of the technology
remedies discussed by Dr. Jones as one step that initiates a
(D) shifting perspective from safety with respect to process that leads to an undesirable end
consumers to safety with respect to producers (C) citing evidence that Dr. Jones lacks the
(E) pointing out an alternative way of obtaining an professional training to judge the case at issue
advantage claimed by the proponent without (D) invoking medical statistics that cast doubt on the
risking a particular disadvantage premises used in Dr. Jones’s argument
(E) providing grounds for dismissing Dr. Jones’s
8. Linguist: Some people have understood certain interpretation of a key term in medical
studies as showing that bilingual children have a technology
reduced “conceptual map” because bilingualism
overstresses the child’s linguistic capacities.
Vocabulary tests taken by bilingual children
appear to show that these children tend to have
a smaller vocabulary than do most children of
the same age group. But these studies are deeply
flawed, since the tests were given in only one
language. Dual-language tests revealed that the
children often expressed a given concept with a
word from only one of their two languages.
The linguist’s argument proceeds by
(A) offering evidence for the advantages of
bilingualism over monolingualism
(B) pointing out an inconsistency in the view that
bilingualism overstresses a child’s linguistic
capabilities
(C) offering evidence that undermines the use of any
vocabulary test to provide information about a
child’s conceptual map
(D) providing a different explanation for the apparent
advantages of bilingualism from the explanation
suggested by the results of certain studies
(E) pointing out a methodological error in the
technique used to obtain the purported evidence
of a problem with bilingualism
10. P: Complying with the new safety regulations is 12. From the tenth century until around the year 1500,
useless. Even if the new regulations had been there were Norse settlers living in Greenland. During
in effect before last year’s laboratory fire, they that time, average yearly temperatures fell slightly
would not have prevented the fire or the injuries worldwide, and some people claim that this temperature
resulting from it because they do not address its drop wiped out the Norse settlements by rendering
underlying causes. Greenland too cold for human habitation. But this
explanation cannot be correct, because Inuit settlers
Q: But any regulations that can potentially prevent
from North America, who were living in Greenland
money from being wasted are useful. If obeyed,
during the time the Norse settlers were there, continued
the new safety regulations will prevent some
to thrive long after 1500.
accidents, and whenever there is an accident here
at the laboratory, money is wasted even if no one Which one of the following is a technique of reasoning
is injured. used in the argument?
Q responds to P’s position by (A) denying the relevance of an analogy
(B) producing evidence that is inconsistent with the
(A) extending the basis for assessing the utility of
claim being opposed
complying with the new regulations
(C) presenting an alternative explanation that purports
(B) citing additional evidence that undermines P’s
to account for more of the known facts
assessment of the extent to which the new
(D) citing a general rule that undermines the claim
regulations would have prevented injuries in last
being opposed
year’s laboratory fire
(E) redefining a term in a way that is favorable to the
(C) giving examples to show that the uselessness
argument’s conclusion
of all regulations cannot validly be inferred
from the uselessness of one particular set of
13. A group of unusual meteorites was found in Shergotty,
regulations
India. Their structure indicates that they originated on
(D) showing that P’s argument depends on the false
one of the geologically active planets, Mercury, Venus,
assumption that compliance with any regulations
or Mars. Because of Mercury’s proximity to the Sun,
that would have prevented last year’s fire would
any material dislodged from that planet’s surface would
be useful
have been captured by the Sun, rather than falling to
(E) pointing out a crucial distinction, overlooked by
Earth as meteorites. Nor could Venus be the source of
P, between potential benefits and actual benefits
the meteorites, because its gravity would have prevented
dislodged material from escaping into space. The
11. Frankie: If jelly makers were given incentives to
meteorites, therefore, probably fell to Earth after being
make a certain percentage of their jellies from
dislodged from Mars, perhaps as the result of a collision
cloudberries, income for cloudberry gatherers
with a large object.
would increase.
The argument derives its conclusion by
Anna: That plan would fail. Cacao, like cloudberries,
was once harvested from wild plants. When (A) offering a counterexample to a theory
chocolate became popular in Europe, the cacao (B) eliminating competing alternative explanations
gathers could not supply enough to meet the (C) contrasting present circumstances with past
increased demand, and farmers began to grow circumstances
large quantities of it at low cost. Now all cacao (D) questioning an assumption
used in commercial chocolate production is (E) abstracting a general principle from specific data
grown on farms. Likewise, if the demand for
cloudberries increases, domesticated berries
grown on farms will completely supplant berries
gathered in the wild.
Anna’s argument proceeds by
(A) giving a reason why a proposed course of action
would be beneficial to all those affected by it
(B) reinterpreting evidence presented in support of a
proposal as a reason to reject the proposal
(C) projecting the result of following a proposal in a
given situation by comparing that situation with
a past situation
(D) proposing a general theory as a way of explaining
a specific market situation
(E) contending that the uses for one product are
similar to the uses for another product
14. Land developer: By attempting to preserve endangered 16. Although Damon had ample time earlier in the month
species that otherwise would become extinct to complete the paper he is scheduled to present at
during our lifetime, we are wasting money on a professional conference tomorrow morning, he
species that will disappear over time regardless repeatedly put off doing it. Damon could still get the
of our efforts. Paleontologists have established paper ready in time, but only if he works on it all
that extinction is the normal fate of species on the evening without interruption. However, his seven-year-
geological time scale of millions of years. old daughter’s tap-dance recital takes place this evening,
and Damon had promised both to attend and to take his
Environmentalist: To claim that we should let species
daughter and her friends out for ice cream afterward.
disappear because all species eventually die out
Thus, because of his procrastination, Damon will be
makes about as much sense as arguing that we
forced to choose between his professional and his family
should not spend money to find a cure for cancer
responsibilities.
because all humans are inevitably mortal.
The argument proceeds by
The method the environmentalist uses to object to the
land developer’s argument is to (A) providing evidence that one event will occur in
order to establish that an alternative event cannot
(A) clarify a dilemma that is embedded in the land
occur
developer’s argument
(B) showing that two situations are similar in order
(B) attack the character of the land developer rather
to justify the claim that someone with certain
than the position the land developer is taking
responsibilities in the first situation has similar
(C) show that more evidence is needed to substantiate
responsibilities in the second situation
the land developer’s conclusion
(C) invoking sympathy for someone who finds
(D) show that the land developer’s line of reasoning
himself in a dilemma in order to excuse that
would lead to an unacceptable conclusion if
person’s failure to meet all of his responsibilities
applied to a different situation
(D) making clear the extent to which someone’s
(E) argue that there are problems that money,
actions resulted in harm to others in order
however judiciously spent, cannot solve
to support the claim that those actions were
irresponsible
15. Party spokesperson: The opposition party’s proposal
(E) demonstrating that two situations cannot both
to stimulate economic activity in the province
occur by showing that something necessary for
by refunding $600 million in provincial taxes to
one of those situations is incompatible with
taxpayers, who could be expected to spend the
something necessary for the other situation
money, envisions an illusory benefit. Since the
province’s budget is required to be in balance,
either new taxes would be needed to make up
the shortfall, in which case the purpose of the
refund would be defeated, or else workers for
the province would be dismissed. So, either the
province’s taxpayers or its workers, who are also
residents of the province, will have the $600
million to spend, but there can be no resulting net
increase in spending to stimulate the province’s
economy.
The spokesperson proceeds by
(A) reinterpreting a term that is central to an opposing
argument
(B) arguing that a predicted advantage would be
offset by an accompanying disadvantage
(C) casting doubt on the motives of opponents
(D) drawing a distinction between different kinds of
economic activity
(E) seeking to show that the assumption that
taxpayers would spend money that might be
refunded to them is dubious
17. Alan: Government subsidies have been proposed 18. A number of seriously interested amateur astronomers
in Cariana to encourage farmers in Rochelle, have tested the new Exodus refractor telescope. With
the country’s principal agricultural region, to it, they were able to observe in crisp detail planetary
implement certain new farming techniques. features that were seen only as fuzzy images in their
Unless these techniques are implemented, erosion 8-inch (approximately 20-centimeter) Newtonian
of productive topsoil cannot be controlled. telescopes, even though the 8-inch telescopes, with
Unfortunately, farmers cannot afford to shoulder their wider apertures, gather more light than the 4-inch
the entire cost of the new techniques, which (approximately 10-centimeter) Exodus. Given these
are more expensive than those currently used. amateur astronomers’ observational findings, any serious
Therefore, without subsidies, agricultural output amateur astronomer ought to choose the Exodus if she
in Rochelle will inevitably decline. or he is buying a telescope for planetary observation.
Betty: But erosion in Rochelle is caused by recurring The argument proceeds by
floods, which will end next year once Cariana
(A) evaluating the credibility of claims made by a
completes the hydroelectric dam it is building
particular group
across the region’s major river. Therefore,
(B) detailing the ways in which a testing situation
Rochelle’s total agricultural output will stabilize
approximates the conditions of ordinary use
at its present level even without subsidies.
(C) placing a phenomenon in a wider context in order
Betty uses which one of the following argumentative to explain it
techniques in countering Alan’s argument? (D) supporting a recommendation to a group on the
basis of the experience of a subset of that group
(A) showing that one premise in Alan’s argument
(E) distinguishing between the actual reasons why a
is inconsistent with another premise in his
certain group did a particular thing and the best
argument
reasons for doing that thing
(B) making additional claims that, if correct,
undermine a premise in Alan’s argument
19. Whittaker: There can be no such thing as the number of
(C) demonstrating that Alan’s conclusion is true but
medical school students who drop out before their
not for the reasons Alan gives to support it
second year, because if they drop out, they never
(D) presenting evidence indicating that the policy
have a second year.
Alan argues in favor of would have damaging
consequences that outweigh its positive Hudson: By your reasoning I cannot help but become
consequences rich, because there is similarly no such thing as
(E) pointing out that Alan’s argument mistakenly my dying before my first million dollars is in the
identifies something as the cause of a trend bank.
when it is really an effect of that trend
Hudson responds to Whittaker by
(A) showing that a relevantly analogous argument
leads to an untenable conclusion
(B) citing a specific example to counter Whittaker’s
general claim
(C) pointing out that Whittaker mistakes a necessary
situation for a possible situation
(D) claiming that what Whittaker says cannot be true
because Whittaker acts as if it were false
(E) showing that Whittaker’s argument relies on
analyzing an extreme and unrepresentative case
20. Wife: The work of the artist who painted the portrait of 21. Walter: For the economically privileged in a society to
my grandparents 50 years ago has become quite tolerate an injustice perpetrated against one of
popular lately, so the portrait has recently become society’s disadvantaged is not just morally wrong
valuable. But since these sorts of artistic fads fade but also shortsighted: a system that inflicts an
rapidly, the practical thing to do would be to sell injustice on a disadvantaged person today can
the portrait while it is still worth something, and equally well inflict that same injustice on a well-
thereby enable our daughter to attend the college to-do person tomorrow.
she has chosen.
Larissa: In our society, the wealthy as well as the
Husband: How could you make such a suggestion? That well-educated can protect themselves against all
painting is the only thing you own that belonged sorts of injustices suffered by the less well-off.
to your grandparents. I don’t think it’s a very Allowing such injustices to persist is bad policy
good painting, but it has great sentimental value. not because it places everyone at equal risk of
Besides, you owe it to our daughter to keep it in injustice but because it is a potent source of social
the family as a link to her family’s past. unrest.
The husband uses which one of the following Larissa responds to Walter by doing which one of the
argumentative techniques in replying to the wife’s following?
suggestion?
(A) giving reason to doubt the truth of Walter’s
(A) taking issue with the practicality of her conclusion
suggestion (B) drawing implausible consequences from Walter’s
(B) questioning her aesthetic judgment assumptions
(C) claiming that the reasons she gives are based on (C) questioning Walter’s authority to address matters
emotions rather than on rational considerations of social policy
(D) asserting that the evidence she cites in support of (D) providing an alternative reason for accepting the
her suggestion is false truth of Walter’s conclusion
(E) invoking a competing obligation that he judges to (E) charging Walter with stopping short of
override her practical considerations recognizing the full implications of his position
22. Gamba: Muñoz claims that the Southwest Hopeville 24. Editorial: Many observers note with dismay the decline
Neighbors Association overwhelmingly opposes in the number of nongovernmental, voluntary
the new water system, citing this as evidence of community organizations. They argue that this
citywide opposition. The association did pass a decline is caused by the corresponding growth
resolution opposing the new water system, but of government services once provided by these
only 25 of 350 members voted, with 10 in favor voluntary community groups. But this may not
of the system. Furthermore, the 15 opposing votes be true. The increase in government services may
represent far less than 1 percent of Hopeville’s coincide with a decrease in volunteerism, but the
population. One should not assume that so former does not necessarily cause the latter; the
few votes represent the view of the majority of latter may indeed cause the former.
Hopeville’s residents.
The editorial undermines the conclusion of the causal
Of the following, which one most accurately describes argument by
Gamba’s strategy of argumentation?
(A) showing that there is no causality involved
(A) questioning a conclusion based on the results of (B) offering a counterexample to the alleged
a vote, on the grounds that people with certain correlation
views are more likely to vote (C) proving that no generalization can properly be
(B) questioning a claim supported by statistical drawn about people’s motives for volunteering
data by arguing that statistical data can be (D) offering an alternate explanation of the
manipulated to support whatever view the correlation cited
interpreter wants to support (E) proving that governments must do what
(C) attempting to refute an argument by showing that, community organizations fail to do
contrary to what has been claimed, the truth of
the premises does not guarantee the truth of the 25. Phoebe: There have been many reported sightings
conclusion of strange glowing lights, but a number of
(D) criticizing a view on the grounds that the view is these sightings have a straightforward, natural
based on evidence that is in principle impossible explanation. They occurred clustered in time
to disconfirm and location around the epicenters of three
(E) attempting to cast doubt on a conclusion by earthquakes, and so were almost certainly
claiming that the statistical sample on which earthquake lights, a form of ball lightning caused
the conclusion is based is too small to be by stresses in the ground.
dependable
Quincy: I am skeptical that the association between the
lights and the earthquakes is anything more than
23. According to the proposed Factory Safety Act, a
a coincidence. The theory that ground stresses
company may operate an automobile factory only if that
factory is registered as a class B factory. In addressing related to earthquakes can cause any kind of
whether a factory may postpone its safety inspections, lightning is extremely speculative.
this Act also stipulates that no factory can be class B In responding to Phoebe, Quincy
without punctual inspections. Thus, under the Factory
Safety Act, a factory that manufactures automobiles (A) takes a correlation to be a causal relation
would not be able to postpone its safety inspections. (B) challenges the accuracy of the data about
sightings that Phoebe takes for granted
The argument proceeds by (C) criticizes Phoebe’s explanation as unsubstantiated
(A) pointing out how two provisions of the (D) offers an explanation of the glowing lights
proposed Factory Safety Act jointly entail the different from Phoebe’s
unacceptability of a certain state of affairs (E) accuses Phoebe of introducing irrelevant
(B) considering two possible interpretations of a information
proposed legal regulation and eliminating the
less plausible one
(C) showing that the terms of the proposed Factory
Safety Act are incompatible with existing
legislation
(D) showing that two different provisions of the
proposed Factory Safety Act conflict and thus
cannot apply to a particular situation
(E) pointing out that if a provision applies in a
specific situation, it must apply in any analogous
situation
26. Yang: Yeast has long been known to be a leaven, that is, 28. Conservationist: The risk to airplane passengers from
a substance used in baking to make breads rise. collisions between airplanes using the airport
Since biblical evidence ties the use of leavens to and birds from the wildlife refuge is negligible.
events dating back to 1200 B.c., we can infer that In the 10 years since the refuge was established,
yeast was already known to be a leaven at that only 20 planes have been damaged in collisions
time. with birds, and no passenger has been injured as
a result of such a collision. The wildlife refuge
Campisi: I find your inference unconvincing; several
therefore poses no safety risk.
leavens other than yeast could have been known
in 1200 B.c. Pilot: You neglect to mention that 17 of those 20
collisions occurred within the past 2 years,
Campisi counters Yang’s argument by
and that the number of birds in the refuge is
(A) suggesting that an alternative set of evidence rapidly increasing. As the number of collisions
better supports Yang’s conclusion between birds and airplanes increases, so does
(B) questioning the truth of a presumption underlying the likelihood that at least one such collision will
Yang’s argument result in passenger injuries.
(C) denying the truth of Yang’s conclusion without
The pilot counters the conservationist by
considering the reason given for that conclusion
(D) pointing out that the premises of Yang’s argument (A) attempting to show that the conservationist’s
more strongly support a contrary conclusion description of the facts is misleading
(E) calling into question the truth of the evidence (B) questioning the conservationist’s motives for
presented in Yang’s argument reaching a certain conclusion
(C) asserting that dangerous situations inevitably
27. Peter: Because the leaves of mildly drought-stressed become more dangerous with the passage of
plants are tougher in texture than the leaves time
of abundantly watered plants, insects prefer to (D) discrediting the moral principle on which the
feed on the leaves of abundantly watered plants. conservationist’s argument is based
Therefore, to minimize crop damage, farmers (E) disputing the accuracy of the figures cited by the
should water crops only just enough to ensure conservationist
that there is no substantial threat, from a lack
of water, to either the growth or the yield of the
crops.
Jennifer: Indeed. In fact, a mildly drought-stressed plant
will divert a small amount of its resources from
normal growth to the development of pesticidal
toxins, but abundantly watered plants will not.
Jennifer’s comment is related to Peter’s argument in
which one of the following ways?
(A) It offers information that supports each of the
claims that Peter makes in his argument.
(B) It supports Peter’s argument by supplying a
premise without which Peter’s conclusion cannot
properly be drawn.
(C) It supports Peter’s argument by offering an
explanation of all of Peter’s premises.
(D) It supports one of Peter’s premises although it
undermines Peter’s conclusion.
(E) It supports the conclusion of Peter’s argument
by offering independent grounds for that
conclusion.
29. Some people claim that the reason herbs are not 31. Anthropologist: After mapping the complete dominance
prescribed as drugs by licensed physicians is that the hierarchy for a troupe of vervet monkeys
medical effectiveness of herbs is seriously in doubt. by examining their pairwise interaction, we
No drug can be offered for sale, however, unless it has successfully predicted more complex forms
regulatory-agency approval for medicinal use in specific of their group behavior by assuming that
illnesses or conditions. It costs about $200 million to each monkey had knowledge of the complete
get regulatory-agency approval for a drug, and only hierarchy. Since our prediction was so accurate,
the holder of a patent can expect to recover such large it follows that the assumption we used to reach it
expenses. Although methods of extracting particular was in fact true.
substances from herbs can be patented, herbs themselves
Primatologist: Although I agree that your assumption
and their medicinal uses cannot be. Therefore, under the
helped you make those predictions, your
current system licensed physicians cannot recommend
conclusion does not follow. You might as well
the medicinal use of herbs.
argue that since we can predict the output of
Which one of the following most accurately describes some bank cash machines by assuming that these
the argumentative technique used in the argument? machines actually want to satisfy the customers’
requests, these cash machines must really have
(A) questioning a claim about why something is the
desires.
case by supplying an alternative explanation
(B) attacking the validity of the data on which a The primatologist uses which one of the following
competing claim is based argumentative techniques in countering the
(C) revealing an inconsistency in the reasoning used anthropologist’s argument?
to develop an opposing position
(A) citing various facts that could not obtain if the
(D) identifying all plausible explanations for why
anthropologist’s conclusion were correct
something is the case and arguing that all but
(B) offering another argument that has as its premise
one of them can be eliminated
the denial of the thesis that the anthropologist
(E) testing a theory by determining the degree to
defends
which a specific situation conforms to the
(C) applying one of the anthropologist’s reasoning
predictions of that theory
steps in another argument in an attempt to show
that it leads to an absurd conclusion
30. Mall manager: By congregating in large groups near
(D) attacking the anthropologist’s expertise by
the stores in our mall, teenagers create an
suggesting the anthropologist is ignorant of the
atmosphere in which many adult shoppers feel
analogy that can be drawn between animals and
uncomfortable. As a result, the adults have begun
machines
to spend less time shopping than they have in the
(E) suggesting that the anthropologist’s argument
past. The mall’s goal in this situation is to prevent
relies on a misinterpretation of a key scientific
a significant loss in overall sales, so merchants
term
should do their utmost to discourage teenagers
from congregating near stores.
Merchant: But the amount spent by teenagers who
congregate near mall stores constitutes a
significant percentage of the total amount spent in
those stores.
The merchant’s response to the manager’s argument is
most accurately described as
(A) disputing the truth of claims the manager offers
as support for the recommendation
(B) giving information that pertains to the relation
between the manager’s recommendation and the
mall’s goal
(C) suggesting that the mall’s goal is an undesirable
one
(D) contending that the manager’s recommendation is
sound but for reasons other than those given by
the manager
(E) using the information cited by the manager to
make an additional recommendation that would
help achieve the goal
32. James: Chemists have recently invented a new 34. Sarah: Some schools seek to foster a habit of
technique for extracting rhodium, an element volunteering in their students by requiring them
necessary for manufacturing catalytic converters to perform community service. But since a
for automobiles, from nuclear waste. Catalytic person who has been forced to do something
converters function to remove noxious gases from has not really volunteered and since the habit of
automobile exhaust. The use of nuclear power volunteering cannot be said to have been fostered
is therefore contributing in at least one way to in a person who has not yet volunteered for
creating a cleaner environment. anything, there is no way this policy can succeed
by itself.
Marta: The technique you mention, though effective, is
still at an experimental stage, so there has been Paul: I disagree. Some students forced to perform
no shift in the sources of the rhodium currently community service have enjoyed it so much
used in manufacturing catalytic converters. that they subsequently actually volunteer to do
something similar. In such cases, the policy
Marta responds to James’s argument by
can clearly be said to have fostered a habit of
(A) casting doubt on the accuracy of the claims made volunteering.
by James in support of his conclusion
Paul responds to Sarah’s argument using which one of
(B) questioning the credibility of advocates of nuclear
the following argumentative techniques?
power
(C) indicating that James is assuming the truth of the (A) He argues that Sarah is assuming just what she
conclusion that he intends to establish sets out to prove.
(D) pointing out a fact that James, in drawing his (B) He argues that Sarah’s conception of what it
conclusion, did not take into account means to volunteer excludes certain activities
(E) point out that James’s premises are no more that ought to be considered instances of
plausible than is his conclusion volunteering.
(C) He introduces considerations that call into
33. Inez: In these poor economic times, people want to be question one of Sarah’s assumptions.
sure they are getting good value for their money. (D) He questions Sarah’s motives for advancing an
I predict people would be more willing to buy argument against the school policy.
antiques at our fair if we first have the object (E) He argues that a policy Sarah fails to consider
inspected by professional appraisers who would could accomplish the same aim as the policy that
remove any objects of questionable authenticity. Sarah considers.
Anika: I disagree with your prediction. Our customers
already are antiques experts. Furthermore, hiring
professional appraisers would push up our costs
considerably, thus forcing us to raise the prices on
all our antiques.
Anika’s response proceeds by
(A) indicating that a particular plan would have an
effect contrary to the anticipated effect
(B) claiming that a particular plan should not be
adopted because, while effective, it would have
at least one undesirable consequence
(C) arguing that an alternative plan could achieve
a desired result more easily than the plan
originally proposed
(D) questioning the assumption that authorities are
available who have special knowledge of the
problem under discussion
(E) offering a counterexample in order to show that a
particular general claim is too broadly stated
35. M: The Greek alphabet must have been invented by 37. Astronomer: Astronomical observations in many areas
some individual who knew the Phoenician writing have become useless because light from nearby
system and who wanted to have some way of cities obscures the stars Many people argue that
recording Homeric epics and thereby preserving since streetlights are needed for safety, such
expressions of a highly developed tradition of interference from lights is inevitable. Here in
oral poetry. Sandsville, however, the local observatory’s
view remains relatively clear, since the city has
P: Your hypothesis is laughable! What would have been
restricted unnecessary lighting and installed
the point of such a person’s writing Homeric
special street lamps that direct all their light
epics down? Surely a person who knew them
downward. It is therefore possible to have both
well enough to write them down would not need
well-lighted streets and relatively dark skies.
to read them; and no one else could read them,
according to your hypothesis. The astronomer’s argument proceeds by
(A) appealing to a scientific authority to challenge a
Which one of the following is an argumentative strategy
widely held belief
that P uses in responding to M?
(B) questioning the accuracy of evidence given in
(A) attacking M’s understanding of the literary value support of the opposing position
of oral poetry (C) proposing an alternative scientific explanation for
(B) disagreeing with M’s thesis without attempting to a natural phenomenon
refute it (D) making a distinction between terms
(C) challenging M’s knowledge of the Phoenician (E) offering a counter example to a general claim
writing system
(D) attempting to undermine M’s hypothesis by 38. Charles: During recessions unemployment typically
making it appear absurd rises. Thus, during a recession air pollution due to
(E) providing an alternative interpretation of evidence automobile exhaust decreases, since fewer people
put forward by M commute in cars to jobs and so cars emitting
pollutants into the air are used less.
36. The widespread staff reductions in a certain region’s
Darla: Why think that air pollution would decrease?
economy are said to be causing people who still have
During a recession fewer people can afford to buy
their jobs to cut back on new purchases as though
new cars, and cars tend to emit more pollutants as
they, too, had become economically distressed.
they get older.
Clearly, however, actual spending by such people
is undiminished, because there has been no unusual Which one of the following most accurately describes
increase in the amount of money held by those people in how Darla’s response is related to Charles’s argument?
savings account. (A) It calls into question the truth of the premises that
The argument in the passage proceeds by doing which Charles uses to support his conclusion.
one of the following? (B) It makes an additional claim that can be true only
if Charles’s conclusion is false.
(A) concluding that since an expected consequence
(C) It presents an additional consideration that
of a supposed development did not occur, that
weakens the support given to Charles’s
development itself did not take place
conclusion by his evidence.
(B) concluding that since only one of the two
(D) It argues that Charles’s conclusion is true,
predictable consequences of a certain kind of
although not for the reasons Charles gives to
behavior is observed to occur, this observed
support that conclusion.
occurrence cannot, in the current situation, be a
(E) It presents an argument showing that the premises
consequence of such behavior
in Charles’ s argument support an absurd
(C) arguing that since people’s economic behavior
conclusion that Charles has overlooked.
is guided by economic self-interest, only
misinformation or error will cause people to
engage in economic behavior that harms them
economically
(D) arguing that since two alternative developments
exhaust all the plausible possibilities, one of
those developments occurred and the other did
not
(E) concluding that since the evidence concerning a
supposed change is ambiguous, it is most likely
that no change is actually taking place
39. Politician: The mandatory jail sentences that became 40. Millions of female bats rear their pups in Bracken Cave.
law two years ago for certain crimes have Although the mothers all leave the cave nightly, on their
enhanced the integrity of our system of justice, return each mother is almost always swiftly reunited
for no longer are there two kinds of justice, the with her own pup. Since the bats’ calls are their only
kind dispensed by lenient judges and the kind means of finding one another, and a bat pup cannot
dispensed by severe ones. distinguish the call of its mother from that of any other
adult bat, it is clear that each mother bat can recognize
Public advocate: But with judges stripped of
the call of her pup.
discretionary powers, there can be no leniency
even where it would be appropriate. So, juries The argument seeks to do which one of the following?
now sometimes acquit a given defendant solely
(A) derive a general conclusion about all members of
because the jurors feel that the mandatory
a group from facts known about representative
sentence would be too harsh. Those juries,
members of that group
then, do not return an accurate verdict on the
(B) establish the validity of one explanation for
defendant’s guilt. This is why it is imperative that
a phenomenon by excluding alternative
the legislation instituting mandatory jail sentences
explanations
be repealed.
(C) support, by describing a suitable mechanism, the
The public advocate responds to the politician’s hypothesis that a certain phenomenon can occur
argument by doing which one of the following? (D) conclude that members of two groups are likely
to share a certain ability because of other
(A) trying to show that the politician’s conclusion
characteristics they share
merely paraphrases the politician’s evidence
(E) demonstrate that a general rule applies in a
(B) claiming that the politician’s evidence, properly
particular case
analyzed, has no bearing on the conclusion the
politician derives from it
(C) arguing that leniency is not a trait of individuals
but that, rather, it is a property of certain kinds
of decisions
(D) arguing that an analysis of the consequences of
certain legislation undermines the politician’s
conclusion
(E) charging that the politician exaggerated the
severity of a problem in order to justify a
sweeping solution
41. Tom: Employers complain that people graduating from 42. Philosopher: The eighteenth-century thesis that motion
high school too often lack the vocational skills is absolute asserts that the change in an object’s
required for full-time employment. Therefore, position over time could be measured without
since these skills are best acquired on the job, reference to the position of any other object. A
we should require high school students to work well-respected physicist, however, claims that this
at part-time jobs so that they acquire the skills thesis is incoherent. Since a thesis that is
needed for today’s job market. incoherent cannot be accepted as a description of
reality, motion cannot be absolute.
Mary: There are already too few part-time jobs for
students who want to work, and simply requiring The argument uses which one of the following
students to work will not create jobs for them. argumentative techniques?
Which one of the following most accurately describes (A) attempting to persuade by the mere use of
how Mary’s response is related to Tom’s argument? technical terminology
(B) using experimental results to justify a change in
(A) It analyzes an undesirable result of undertaking definition
the course of action that Tom recommends. (C) relying on the authority of an expert to support a
(B) It argues that Tom has mistaken an unavoidable premise
trend for an avoidable one. (D) inferring from what has been observed to be the
(C) It provides information that is inconsistent with case under experimental conditions to what is in
an explicitly stated premise in Tom’s argument. principle true
(D) It presents a consideration that undercuts an (E) generalizing from what is true in one region of
assumption on which Tom’s argument depends. space to what must be true in all regions of
(E) It defends an alternative solution to the problem space
that Tom describes.
43. Lydia: Red squirrels are known to make holes in the 45. Essayist: Knowledge has been defined as a true belief
bark of sugar maple trees and to consume the formed by a reliable process. This definition has
trees’ sap. Since sugar maple sap is essentially been criticized on the grounds that if someone
water with a small concentration of sugar, the had a reliable power of clairvoyance, we would
squirrels almost certainly are after either water not accept that person’s claim to know certain
or sugar. Water is easily available from other things on the basis of this power. I agree that we
sources in places where maple trees grow, so the would reject such claims, but we would do so
squirrels would not go to the trouble of chewing because we really do not believe in clairvoyance
holes in trees just to get water. Therefore, they are as a reliable process. Were we to believe in
probably after the sugar. clairvoyance, we would accept knowledge claims
made on the basis of it.
Galina: It must be something other than sugar, because
the concentration of sugar in the maple sap is Which one of the following most accurately describes
so low that a squirrel would need to drink an the essayist’s method of defending the definition against
enormous amount of sap to get any significant the objection?
amount of sugar.
(A) asserting that the objection is based on a belief
Lydia’s argument proceeds by about the reliability of clairvoyance rather than
on the nature of knowledge or its definition
(A) dismissing potentially disconfirming data
(B) asserting that the case of clairvoyance is one of
(B) citing a general rule of which the conclusion is a
knowledge even though we do not really believe
specific instance
in clairvoyance as a reliable process
(C) presenting an observed action as part of a larger
(C) arguing against the assumption that clairvoyance
pattern of behavior
is unreliable
(D) drawing an analogy between well-understood
(D) explaining that the definition of knowledge is a
phenomena and an unexplained phenomenon
matter of personal choice
(E) rejecting a possible alternative explanation for an
(E) demonstrating that the case of clairvoyance is
observed phenomenon
not a case of knowledge and does not fit the
definition of knowledge
44. It is widely believed that eating chocolate can cause
acne. Indeed, many people who are susceptible to acne
report that, in their own experience, eating large
amounts of chocolate is invariably followed by an
outbreak of that skin condition. However, it is likely that
common wisdom has mistaken an effect for a cause.
Several recent scientific studies indicate that hormonal
changes associated with stress can cause acne and there
is good evidence that people who are fond of chocolate
tend to eat more chocolate when they are under stress.
The argument employs which one of the following
argumentative strategies?
(A) It cites counterevidence that calls into question
the accuracy of the evidence advanced in support
of the position being challenged.
(B) It provides additional evidence that points to an
alternative interpretation of the evidence offered
in support of the position being challenged.
(C) It invokes the superior authority of science over
common opinion in order to dismiss out of hand
the relevance of evidence based on everyday
experience.
(D) It demonstrates that the position being challenged
is inconsistent with certain well-established
facts.
(E) It provides counterexamples to show that,
contrary to the assumption on which the
commonly held position rests, causes do not
always precede their effects.
46. Hospital auditor: The Rodríguez family stipulated that 47. Dillworth: More and more people are deciding
the funds they donated to the neurological clinic not to have children because of the personal
all be used to minimize patients’ suffering. The and economic sacrifices children require and
clinic administration is clearly violating those because so often children are ungrateful for the
terms, since it has allocated nearly one fifth of considerable sacrifices their parents do make
those funds for research into new diagnostic for them. However, such considerations have no
technologies, instead of letting that money flow bearing on the fact that their children provide
directly to its patients. the best chance most people have of ensuring
that their values live on after them. Therefore,
Clinic administrator: But the successful development
for anyone with deeply held values, foregoing
of new technologies will allow early diagnosis
parenthood out of reluctance to make sacrifices
of many neurological disorders. In most cases,
for which little gratitude can be expected would
patients who are treated in the early stages
probably be a mistake.
of neurological disorders suffer far less than
do patients who are not treated until their Travers: Your reasoning ignores another fact that
neurological disorders reach advanced stages. deserves consideration: children’s ingratitude for
parental sacrifices usually stems from a wholesale
The clinic administrator responds to the hospital auditor
rejection of parental values.
by doing which one of the following?
Dillworth employs which one of the following
(A) demonstrating that the hospital auditor’s
argumentative strategies?
conclusion, though broadly correct, stands in
need of a minor qualification (A) showing that considerations cited as drawbacks to
(B) showing that the hospital auditor’s argument fails a given course of action are not really drawbacks
to separate what is the case from what ought to at all
be the case (B) exposing as morally suspect the motives of
(C) reminding the hospital auditor that, in the case people who would make the choice that
at issue, being told what to do is tantamount to Dillworth rejects
being told how to do it (C) indirectly establishing that a given course
(D) arguing that, in assessing the severity of a of action is obligatory by arguing that the
violation, the reasoning motivating the violation alternative course of action is prohibited
needs to be considered (D) distinguishing a category of person for whom
(E) reinterpreting a key phrase in the hospital the reason presented in favor of a given course
auditor’s argument so as to undermine an of action is more telling than the reasons cited
assumption underlying that argument against that course of action
(E) using evidence that a certain course of action
would be appropriate under one set of conditions
to arrive at a general conclusion about what
would be appropriate in all cases
48. Philosopher: It is absurd to argue that people are 50. Statistician: Changes in the Sun’s luminosity correlate
morally obligated to act in a certain way exceedingly well with average land temperatures
simply because not acting in that way would on Earth. Clearly—and contrary to accepted
be unnatural. An unnatural action is either a opinion among meteorologists—the Sun’s
violation of the laws of nature or a statistical luminosity essentially controls land temperatures
anomaly. There is no possibility of acting as one on Earth.
cannot, nor does the mere fact that something is
Meteorologist: I disagree. Any professional
not usually done provide any good reason not to
meteorologist will tell you that in a system as
do it.
complicated as that giving rise to the climate, no
Which one of the following most accurately describes a significant aspect can be controlled by a single
technique used in the philosopher’s argument? variable.
(A) undermining a concept by showing that its The rejection by the meteorologist of the statistician’s
acceptance would violate a law of nature conclusion employs which one of the following
(B) stating the definition of a key term of the techniques of argumentation?
argument
(A) supporting a conclusion about a specific case by
(C) using statistical findings to dispute a claim
invoking a relevant generalization
(D) undermining a claim by showing that the claim is
(B) producing a single counterexample that
self-contradictory
establishes that a generalization is false as stated
(E) using empirical evidence to support one definition
(C) reanalyzing a correlation as reflecting the
of a key term of the argument over another
multiple effects of a single cause
(D) rejecting a conclusion because it is a proposition
49. Zachary: The term “fresco” refers to paint that has
that cannot be experimentally tested
been applied to wet plaster. Once dried, a fresco
(E) pointing out that potentially unfavorable evidence
indelibly preserves the paint that a painter has
has been systematically neglected
applied in this way. Unfortunately, additions
known to have been made by later painters
51. Some people have been promoting a new herbal mixture
have obscured the original fresco work done by
as a remedy for the common cold. The mixture contains,
Michelangelo in the Sistine Chapel. Therefore, in
among other things, extracts of the plants purple
order to restore Michelangelo’s Sistine Chapel
coneflower and goldenseal. A cold sufferer, skeptical of
paintings to the appearance that Michelangelo
the claim that the mixture is an effective cold remedy,
intended them to have, everything except the
argued, “Suppose that the mixture were an effective
original fresco work must be stripped away.
cold remedy. Since most people with colds wish to
Stephen: But it was extremely common for painters of recover quickly, it follows that almost everybody with a
Michelangelo’s era to add painted details to their cold would be using it. Therefore, since there are many
own fresco work after the frescos had dried. people who have colds but do not use the mixture, it is
obviously not effective.”
Stephen’s response to Zachary proceeds by
Which one of the following most accurately describes
(A) calling into question an assumption on which
the method of reasoning the cold sufferer uses to reach
Zachary’s conclusion depends
the conclusion of the argument?
(B) challenging the definition of a key term in
Zachary’s argument (A) finding a claim to be false on the grounds that it
(C) drawing a conclusion other than the one that would if true have consequences that are false
Zachary reaches (B) accepting a claim on the basis of public opinion
(D) denying the truth of one of the stated premises of of the claim
Zachary’s argument (C) showing that conditions necessary to establish the
(E) demonstrating the Zachary’s conclusion is not truth of a claim are met
consistent with the premises he uses to support it (D) basing a generalization on a representative group
of instances
(E) showing that a measure claimed to be effective in
achieving a certain effect would actually make
achieving the effect more difficult
52. S: It would be premature to act to halt the threatened 53. Psychologists have claimed that many people are more
“global warming trend,” since that alleged trend susceptible to psychological problems in the winter than
might not be real. After all, scientists disagree in the summer; the psychologists call this condition
about it, some predicting over twice as much seasonal affective disorder. Their claim is based on the
warming as others, so clearly their predictions results of surveys in which people were asked to recall
cannot be based on firm evidence. how they felt at various times in the past. However, it
is not clear that people are able to report accurately on
W: Most scientists consider discussions of accepted
their past psychological states. Therefore, these survey
ideas boring, and prefer to argue about what is
results do not justify the psychologists’ claim that there
not known. According to the International Science
is any such condition as seasonal affective disorder.
Council, there is a consensus among reputable
investigators that average global warming in the The author criticizes the psychologists’ claim by
next century will be from 1.5° to 4.5°C.
(A) offering an alternative explanation of the variation
W’s rejoinder proceeds by in the occurrence of psychological problems
across seasons
(A) denying the existence of the disagreements cited
(B) questioning whether any seasonal variation in
by S
the occurrence of psychological problems could
(B) accepting S’s conclusion while disputing the
properly be labeled a disorder
reasons offered for it
(C) questioning the representativeness of the
(C) relying on authorities whose views conflict with
population sample surveyed by the psychologists
the views of the authorities cited by S
(D) questioning an assumption that the author
(D) putting disagreements cited by S in perspective
attributes to the psychologists
by emphasizing similarities
(E) demonstrating that fewer people actually
(E) reasoning in a circle by accepting evidence only
suffer from seasonal affective disorder than
if it agrees with a desired conclusion
psychologists had previously thought
46. E
1. C
47. D
2. C
48. B
3. C
49. A
4. B
50. A
5. A
51. A
6. C
52. D
7. E
53. D
8. E
9. B
10. A
11. C
12. B
13. B
14. D
15. B
16. E
17. B
18. D
19. A
20. E
21. D
22. E
23. A
24. D
25. C
26. B
27. E
28. A
29. A
30. B
31. C
32. D
33. A
34. C
35. D
36. A
37. E
38. C
39. D
40. B
41. D
42. C
43. E
44. B
45. A
she doesn’t present an alternative suggestion (B). (E) seems
to imply that Aiesha is supplying additional evidence in accord
with Adam’s evidence, when she is actually providing evidence
to contradict Adam.
1. (C)
By summarizing each person’s point of view, you can 4. (B)
characterize how one responds to the other. Sheila is worried that putting up signs and speed bumps is
Xavier’s purpose in proposing that Nepal prohibit tourist likely to cause serious accidents when speeders hit the speed
sales of thangkas is to encourage young artists to create the bumps. In other words, Robert’s solution is likely to have
traditional ones rather than cheap knockoffs. But if Nepal serious new side effects (B).
does so, Yvette replies, thangkas won’t be made at all—just as Sheila certainly never implies that speeding isn’t a serious
correct choice (C) describes. problem, so we can rule out (A). (C) and (D) are also out, since
(A) Yvette’s concern with the decay and death of Nepalese art Sheila only attacks Robert’s suggestion, and never suggests
shows that she’s just as cognizant of the problem as Xavier is. any alternative course of action (C) or endorses Robert’s
(B) , (D) Yvette presents a necessary condition for healthy art suggestion for alternate reasons (D). (E) might be tempting,
and then speculates on what would happen if Xavier’s plan but beware of the phrase “no net effect.” Sheila doesn’t just
think Robert’s suggestion would be ineffective—she feels it
went through. None of that challenges his information (B) or
would actually cause new accidents. We have no information
makes any analogy (D).
about whether Sheila feels the “net” effect would be more or
(E) Yvette accepts Xavier’s evidence; she just feels his fewer accidents, but since she describes it as a bad idea, she
proposed solution would intensify the problem. certainly doesn’t simply regard it as having “no net effect.”
2. (C) 5. (A)
Carla says, “But how do you go about choosing which Joanna says that a company emerging from bankruptcy must
perspective is the valid one? . . . Your approach would generate not (“the only way”) change its business. Yet Ruth is quick to
a biased version of history .... ” This is nicely summed up point to such a company that diversifies and succeeds in a
in (C). way that Joanna deems “futile.” Simply put, that’s a “counter”
Although Carla says that Mark’s proposed method of history to Joanna’s “claim,” (A).
would “generate a biased version of history,” she never Ruth “explains” (B) how the Kelton Co. turned itself around,
indicates that such a method would be impossible (B), nor but that’s not an “alternative” to any “phenomenon” Joanna
does she actually question Mark’s own understanding of describes. An analogy (C) is a declaration that two unlike
historical events (A). (D) uses the phrase “the kind of writing things share some significant similarity; but Kelton is an
[Mark] deplores,” but there is no indication that Mark deplores example of a once-bankrupt company, not an analogy to one.
anything. (E) is also way off base, introducing “professional Both Joanna and Ruth’s claims are concrete, and neither
self-interest.” accuses the other of ambiguity (D). And if anyone here is
“excluding a plausible alternative” (E) it’s Joanna, who
3. (C) refuses to accept that a company can change its nature after
Adam assumes that “enabling [drivers] to drive more safely” bankruptcy and become successful.
will “decrease the annual number of accidents.” That’s a
reasonable assumption, but Aiesha points out an alternative 6. (C)
outcome—clearer markings will actually make it easier for (C) has it right: The author takes the very evidence (few and
people to drive faster and closer to the edge of the road, tiny strikes) used by the paper to belittle unions, and shows
behaviors that causes accidents. Aiesha is thus pointing out that it can speak to unions’ strength.
the sad fact that making it easier for people to drive safely will
Far from questioning the evidence’s accuracy (A) the
not necessarily ensure that people do so (C).
author accepts it and uses it to his own ends. The author’s
Aiesha disagrees that reflecting posts will decrease accidents, evidence is interpretive rather than historical (B) and no
but she doesn’t disagree that they will make the road edge hint of “outdatedness” appears. In questioning the article’s
easier to see (A), or that they are relevant to the number conclusion, the author does not impugn its writer’s motives
of accidents (D) (though she thinks they will increase, not (D) nor does he need to; (E) seems to put its toe in the water
decrease, that number). Although she disagrees with Adam, of impugning motives as well.
7. (E) of telemedicine will turn into debits. There’s no personal
What the proponent propounds is the subjecting of food to criticism that amounts to an ad hominem attack, (C), nor are
gamma irradiation which, he asserts, retards spoilage, doesn’t any statistics cited whatsoever, (D). And while Carabella takes
compromise nutrition or safety, and kills Salmonella. The strong issue with Jones’s optimism, she does not challenge
opponent in effect says, hey, we should employ this chemical any of his terminology, (E).
dip, which will not only kill Salmonella but will also avoid
a disadvantage of irradiation (namely that irradiation both 10. (A)
leaves botulism bacteria unscathed and eliminates a botulism So, what’s Q’s method of argument? You should have had
warning). The above states in specific terms that which (E) a pretty good feel for this, whether or not you were able to
asserts in abstract ones. prephrase it precisely. The best way to go here is probably to
Both arguments are quite concrete and leave no room for knock off the things she doesn’t do on our way to the choice
verbal ambiguity (A). That the opponent contradicts the that sounds like a reasonable description of her method.
proponent is not the same as saying that he proves a self- (A) sounds pretty darn reasonable—she does broaden the
contradiction (B). Besides irradiation, the only other proposed concept of utility by bringing up the money issue whereas P
remedy mentioned (C) is the chemical dip, about which the spoke only of preventing injuries. It makes it easier to debate
opponent establishes no undesirable consequences. Contrary P on the usefulness of the regulations when she extends the
to (D), the opponent is exclusively concerned with consumer argument in this way. Let’s keep (A) on hold just to scan the
safety and not at all with producer safety. rest of the choices to make sure they’re as wrong as they need
to be:
8. (E) (B) Q doesn’t attack on this front—she never argues that the
“Abstracting” an argument means reducing each sentence to regulations would have prevented injuries from last year’s fire.
its purpose. We do it a lot in Parallel Reasoning, and it comes Instead, she switches the focus to saving money.
in handy in Method questions too. Sentence 1: allegation (C) is way out there. First of all, she doesn’t concede the
(studies show that bilingual kids have a reduced “conceptual uselessness of these regulations—quite the opposite, in fact—
map,” whatever that means; the important thing is that it nor does she argue against the position that all regulations are
sounds bad). Sentence 2: some detail about those studies useless.
(i.e., that which they “appear to show”). Sentence 3: author’s
(D) P is not concerned with any regulations that would have
value judgment (the studies are flawed). Sentence 4: the truth
prevented last year’s fire—his point is that these specific new
behind the test results. Since the author’s overall purpose
regulations wouldn’t have done the job. Q doesn’t expose any
is clearly to discredit this rap at bilingualism—and since she
underlying assumption on P’s part regarding what he would
does so by pointing out where the vocabulary tests went
consider to be useful regulations.
wrong—(E) must be the right answer.
(E) Again, there’s no indication that Q is directly confronting
(A) and (D) exaggerate the author’s defense of bilingualism
any error on P’s part. She’s simply bringing up a new
into an effort to demonstrate its superiority, but the author
consideration that she presumably hopes could change P’s
doesn’t make that claim and the evidence doesn’t lead to
view of the new regulations. She doesn’t state or imply that P’s
it. The view she’s trying to rebut (B) is that bilingual kids are
argument is lacking something fundamental; only that another
verbally underdeveloped, so (B) is a 180, and she points to a
consideration may also be relevant. (E)’s vague “difference
misinterpretation, not an “inconsistency,” so (B) is an overall
between potential and actual benefits” plays no part in Q’s
train wreck. (C) leaves out bilingualism, which is at the heart
response to P’s argument.
of the author’s interest, and implies that her purpose is to
skewer vocabulary tests in general. Nothing good among these choices, which confirms our
suspicion that (A)’s the one to choose here.
9. (B)
11. (C)
“Which of the following strategies . . .” signals a Method
question; form a solid prephrase before attacking the choices. It’s Anna we’re concerned with here, as the stem helpfully
points out—specifically, her method of argument. Only a gong
Carabella’s sheer movement from “initially” to “eventually,”
or buzzer would state her opinion of Frankie’s plan more
and her dire predictions as to the ultimate bitter end of what
emphatically than does her first sentence—the plan’s a loser.
at first seems to be a smart technology, should make (B)
Frankie believes that if it could be made worth their while for
stand out.
jellymakers to use cloudberries, cloudberry gatherers would
Carabella never claims that a seeming benefit is in fact make more money. But Anna rains on that parade by bringing
harmful, (A), though she does feel that the initial benefits up the subject of cacao, a product also once harvested in the
wild. When demand for it rose, she claims, cacao began to be alternative explanation. Thus, the facts are left cooling on the
grown cheaply on farms, to the point where no commercial page, and (C) doesn’t work.
production of it relied on cacao gatherers. And here’s her (D) What is the rule? The author cites no general rule. A general
point, signaled blatantly by the Keyword “Likewise”: If rule would be something like “Greenland can never be too
cloudberry demand rises, the same thing will happen, and cold for human habitation.” But the author doesn’t go this
cloudberry gatherers, far from benefiting economically, would route—instead, he states a specific example (the survival of
be out of luck. Her argument relies on a comparison to a the Inuit) that he uses to refute the opposing claim.
similar previous situation, a situation that turned out in a
(E) Another complete miss. The author doesn’t redefine any
way which suggests that Frankie’s plan’s off-base. This is as
terms. Cold is cold, Norse is Norse, Inuit is Inuit, Greenland is
good a pre-phrased notion as any, and every element of (C)
Greenland, and so on. If you chose (E), look back to see what
corresponds to it: The proposal for the cloudberry situation
term you feel was redefined along the way in the author’s
is compared with what happened in the previous cacao
favor. Was it really?
situation, and a result of the cloudberry plan is projected—
failure, hands down.
13. (B)
(A) Anna actually suggests the opposite, at least in regards to
Since we’re asked how the author puts together her argument,
cloudberry gatherers; that group would be presumably hurt by
we need to identify the author’s Method of Argument. The
the plan.
argument proceeds in a linear fashion: Meteors came from
(B) What evidence is reinterpreted? Frankie’s argument is a either Mercury, Venus, or Mars. The source wasn’t Mercury
simple if-then statement: If X happens, then Y will happen. (too close to the Sun), and it wasn’t Venus (gravity problems),
Anna doesn’t tinker with any specific piece of evidence—in and so, the author announces, the source must be Mars. We
fact, there’s not much to tinker with. She merely says “If X don’t learn anything else about Mars, but we don’t need to, as
happens, Y will NOT happen.” The reason for her judgment long as the only candidates are those three planets and the
comes from comparing the proposal to a seemingly similar other two are out. So, the author came to her conclusion by
case. eliminating the competing explanations, as (B) points out.
(D) is a major-league distortion—Anna doesn’t propose (A) No particular examples are cited, and the author isn’t
anything at all. She’s in the business of shooting down disputing one theory; she’s supporting a theory by debunking
proposals. its alternatives.
(E) contains a tricky subtle distortion, not nearly as blatant (C) What contrast? The story describes what must have been
as (D)’s: There is, according to Anna, a similarity between the the case, based on what is known about the planets. No
respective situations of the cloudberry and cacao industries; contrast between past and present is made.
specifically in regard to what happens to independent
(D) The author doesn’t critique an argument by questioning an
gatherers when demand for such products rises. Nowhere,
assumption; the only view presented is her own.
however, does she argue that there’s a similarity in the uses of
these products. That would be a different argument altogether. (E) What general principle? The author’s conclusion is limited
to these specific meteorites.
12. (B)
14. (D)
The writer argues that blaming inhuman cold for the
disappearance of the Norse from Greenland doesn’t work The question stem alerts us to look for the method used by
because another group, the Inuit, thrived there even as the the environmentalist in objecting to the land developer’s
Norse disappeared. In more generic terms, the author rejects position. That is, we want to know how the environmentalist
the original conclusion (that cold killed off the Norse) after argues, not what the argument is. The developer’s main point
considering additional evidence (the survival of the Inuit is that it’s a waste of money to try to preserve species that will
during the same time period). This method of argument is soon become extinct anyway. How does the environmentalist
described in (B). argue against this point? The environmentalist responds
by comparing this reasoning to not spending money on
(A) The author doesn’t present an analogy to refute, so (A)
cancer research just because everyone must eventually
cannot be the method of argument. An analogy is a likening
die. The environmentalist strongly implies that it would be
of the mechanism of one phenomenon or event to another
unheard of to take this position with respect to humans.
seemingly unrelated phenomenon or event; and no such thing
The environmentalist brings up a totally different situation
happens here.
in order to show that it would be reprehensible to espouse
(C) The author explicitly states that the inhuman-cold the developer’s philosophy in other circumstances. Only one
explanation doesn’t suffice, but then fails to offer an
choice even mentions applying the developer’s logic to a how the province will recoup the tax refund, but these are not
separate situation, and that’s (D). “economic activities.”
(A) A dilemma is a choice between equally unsatisfactory (E) The spokesperson attacks the notion that the province
alternatives, resulting in a seemingly-unresolvable situation. will see a net increase in spending by refunding the taxes
There is no such thing in the developer’s argument; the by offering an explanation of how the lost money would be
developer clearly believes that we should let animals become compensated for in the budget. While the author could have
extinct and pocket the cash. claimed that the tax break won’t lead to any more spending,
(B) The environmentalist does not say negative things about that argument isn’t present here.
the developer, which is what would have to be going on here
for this answer to be correct. Instead, the environmentalist 16. (E)
attacks the acceptability or appropriateness of the developer’s Here the choices are ever so much denser than the situation,
argument. which is pretty simple. Damon cannot attend the recital and
(C) The environmentalist seeks to discredit the developer’s go out afterwards and finish the paper in time. The need for
argument by showing how wrong it would be when 100% uninterrupted effort to finish the paper is the necessary
applied to humans. There is no call for more evidence. The condition about which (E) is speaking. Because he must stay
environmentalist dismisses the developer’s conclusion home, he cannot go out.
unequivocally. (A) distorts the situation badly. Contrary to (A), the author
(E) The environmentalist is not concerned, as this answer doesn’t predict which choice Damon will make (to finish the
suggests, with whether or not money can solve problems. paper or go to the recital); she simply argues that he cannot
This is a distortion of the environmentalist’s reference to not do both.
funding cancer research. The point of the reference is to show (B) is an even more egregious distortion. No essential
how wrong the developer’s position is, not to make a point similarities between the work and family situations are
about the power of, or, more accurately, the lack of power of described, and there’s no claim of similar responsibilities that
money. needs “justifying.”
(C) The author demonstrates no sympathy for Damon, and no
15. (B) interest whatsoever in excusing him. Damon’s dilemma is of
The stem asks us to find the choice that describes how the his own making.
argument proceeds, so this is another Method of Argument (D) While the author does seem to find Damon’s self-created
question. The spokesperson asserts that the tax refund will dilemma somewhat blameworthy, “irresponsible” is too strong
not result in a net increase in spending since other taxes a judgment on the evidence provided. Also, what “harm to
would have to be raised or province employees must lose others” is described?
their jobs to compensate for the lost revenue. Even though the
taxpayers received a refund, either all of the taxpayers must 17. (B)
pay more taxes and won’t have the extra money to spend, or
Alan concluded that subsidies are necessary to prevent a
some province employees will lose their jobs and not have any
decline in agricultural output. He came to this conclusion
money to spend. If one of these two scenarios must occur, as
based on two main premises: that soil erosion can’t be
the spokesperson contends, then the refund will not result in
controlled without the implementation of new techniques
more money being spent in the province. In both scenarios,
and that farmers can’t implement the techniques without
the province offsets the refund by imposing a financial penalty
subsidies. In order to disprove this conclusion, Betty
on at least some of the people. Or as (B) puts it, an expected
introduces a new factor, the hydroelectric dam. She claims
advantage is offset by a disadvantage.
that floods cause erosion, and that the dam will prevent the
(A) No, the spokesperson does not reinterpret any term. No floods (and thus control the erosion). Thus, as (B) says, she
terms are singled out at all. uses new claims (the dam will end flooding) to undermine
(C) This argument attempts to refute the issue of the tax Alan’s premise that the new farming techniques are necessary
refund by explaining how it won’t result in added income. No to control soil erosion.
personal attack on the opposition occurs. Other than stating (A) Betty doesn’t try to show an internal contradiction in
that the proposal is the opposition party’s plan, no mention is Alan’s argument, but instead introduces an entirely new
made at all of the opponents themselves. consideration.
(D) No distinction is made between different types of (C) can’t be accurate, since Betty clearly disagrees with Alan’s
economic activity. Two possible scenarios are provided for conclusion, as evidenced by the last sentence of her response.
(D) Betty doesn’t show that the implementation of the new possible or unrepresentative case). In fact, Hudson does not
techniques will have bad consequences; rather, she argues discuss the way in which Whittaker makes her argument at all.
that their implementation simply isn’t necessary to control (D) Hudson does not make a claim or any other affirmative
erosion. statement about the way in Whittaker makes her argument.
(E) No, Betty doesn’t deny that soil erosion is at the root of Hudson’s response is indirect (an analogy).
declining output—she simply offers an overlooked cause for
the erosion. If there’s any “cause” and “trend” here, it’s that 20. (E)
erosion will cause a trend of falling output. On this, Betty and In response to his wife’s desire to sell a painting in order
Alan seem to agree. But (E) asserts that Betty thinks Alan is to pay for their daughter’s college education, the husband
mixing these things up—that is, that he’s saying that declining offers three different reasons for hanging on to the thing, but
output causes erosion. Betty, of course, never suggests such it’s the third reason that relates most to her argument: the
a thing. husband sees the family link as an obligation that overrides
the practical consideration of paying for college.
18. (D)
(A) and (D) The husband doesn’t argue that her solution is
This ad for Exodus telescopes is directed at “any serious either impractical or false: he doesn’t argue that the painting
amateur astronomer,” and makes its appeal based on the nifty won’t pay for college, only that that shouldn’t be the governing
findings of other serious amateur astronomers—a clear-cut consideration.
case of appealing to a group based on the experience of a
(B) Since the wife doesn’t say that the painting is lovely (just
subset.
that it’s valuable), the husband’s criticism is not a slur on her
(A) Astronomers’ claims are reported but not evaluated as to taste.
credibility. Indeed, they are taken on faith.
(C) If anything, it’s the husband whose recommendation is
(B) suggests a dichotomy—between testing conditions and based on emotion—sentimental value and family connections
“ordinary” conditions—not supported by the passage. The ad and all that—not the wife.
seems to assume that the large group of astronomers will work
with the Exodus in the same way, and have the same success, 21. (D)
that the subgroup did.
Walter’s conclusion is that tolerating an injustice is morally
(C) is just gibberish. There’s no “wide context” here, just wrong and shortsighted. His evidence follows the colon
some specific test results by amateur astronomers that (it is (always pay attention to punctuation). Larissa also believes
suggested) are replicable by others. that allowing injustices is bad policy, but disagrees with Walter
(E) Motive never enters into it. Presumably the motives are as to the reason. The key to understanding Larissa’s evidence
self-evident and clear-cut, i.e., observing the planets and is the phrase “not because . . . but because.” She disagrees
trying out the Exodus. with Walter’s reason for the conclusion and provides her own.
Therefore, you’re looking for an answer choice, such as (D),
19. (A) which states that Larissa agrees with Walter’s conclusion but
Hudson does not respond directly to Whittaker’s argument. disagrees with his evidence (reason).
Instead, he applies Whittaker’s reasoning— “by your (A) might have been tricky if you didn’t read the stimulus
reasoning”—to an entirely different situation. A big clue that critically. Larissa agrees with Walter’s conclusion, she only
Hudson is applying Whittaker’s logic to a different situation disagrees with his reasoning.
is his use of the word “similarly.” Hudson claims, based on (B) Larissa’s conclusion is the same as Walter’s, and she
Whittaker’s reasoning, that it is inevitable that he will become bases it on her own evidence—she doesn’t do anything with
rich because he cannot die before he has his first million any assumptions Walter may have, let alone draw implausible
dollars in the bank! Now, it’s certainly possible, if not likely, consequences from them. (She does disagree with his
that you can die before you have your first million in the bank. evidence regarding the wealthy’s vulnerability to injustice, but
And not having your first million in the bank is very unlikely to that’s another matter.)
prevent you from dying. Thus, Hudson responds to Whittaker
(C) Larissa deals only with Walter’s argument; she never
by making an analogy that leads to a silly conclusion—
questions Walter himself or his authority to address matters
choice (A).
of social policy. In order for (C) to be correct, Larissa would
(B) Hudson does not provide a specific example (for instance, need to have said something like “Walter is not in a position
a specific medical student) to counter Whittaker’s claim. to make his assertions because he is neither an economist nor
(C) and (E) Hudson does not respond to Whittaker by pointing a politician.”
out/showing a flaw in Whittaker’s reasoning (necessary vs.
(E) Although Larissa believes that the possibility of social must have punctual inspections, and the argument concludes
unrest is the reason that we should not tolerate injustice, that the Factory Safety Act would prevent auto factories
she does not mention a belief that Walter should have more from postponing inspections. This is a classic Formal Logic
fully developed his argument. She simply disagrees with his formulation: If A, then B; If B, then C, therefore, if A, then C.
rationale. That’s what we’ll look for in the choices, and we find it in (A):
two provisions of the Factory Safety act combine to produce a
22. (E) certain result.
An author will often proceed by pointing out a flaw in a (B) There’s only one interpretation of the Factory Safety Act
position she opposes. given in the stimulus, not two.
Whenever the GMAT begins with the formulation “so-and-so (C) No existing legislation is mentioned in the stimulus.
claims that . . .” you should expect the author to contradict the (D) The two provisions of the proposed Factory Safety Act don’t
claim somehow. After all, if they weren’t going to contradict conflict—in fact, they combine to produce a certain result.
a claim, they’d make it themselves, instead of pointing out
(E) There’s no analogy in the stimulus. The provision in
who is responsible for such a ridiculous idea. This argument is
question is only discussed as it relates to a single specific
no different. Gamba begins by outlining Muñoz’s claims: that
situation.
the Hopewell Neighbors Association opposes the new water
system, and that this means the city opposes it. She then
24. (D)
goes on to shred Muñoz’s evidence for his claims. As Gamba
tells us, it turns out that less than 10% of the 350-member Remember the three classic alternatives to causal arguments.
Association even voted on the water system, and that even We know from the question stem that we’ll be dealing with a
then the vote was pretty close—it came out 15 to 10 against causal argument: X caused Y. You should immediately think of
the water system. The 15 opposing votes are less than 1% of the three classic alternatives: Y caused X; Z caused Y and/or X;
the membership of the association, and Gamba tells us that or the link was a coincidence.
such a small sample doesn’t accurately represent the views Here, the author is the one weakening a causal argument, so
of the association, much less the town as a whole. If the word we’ll see if he uses one of these three. Sure enough, the last
“represent” in the conclusion turned on a light bulb in your sentence, “the latter may indeed cause the former,” tells us
brain that said, “representativeness,” you probably got this that we’re looking for “Y caused X,” a reversal of the causal
question correct very quickly. (E) recognizes that Gamba points argument. We find this in (D).
out the flaw of representativeness in Muñoz’s argument.
(A) contradicts the stimulus—the author tells us that the
(A) Perhaps the reason that so few members of the Association causal relationship is reversed, not nonexistent.
voted on the water measure is that people with certain views
(B) There is no counterexample cited.
were more likely to vote, as (A) suggests, but Gamba never
brings this up in her argument. (C) “People’s motives for volunteering” are not part of the
argument; we only know that fewer people are volunteering,
(B) Gamba also never mentions the possibility of statistical
not why.
manipulation.
(E) may be an argument for why the government should take
(C) Gamba never gets down to arguing about whether the
over community services, but doesn’t speak to the causal link.
truth of the premises guarantees the truth of the conclusion—
she denies the truth of the premise Muñoz uses, that the
25. (C)
Neighbors’ Association overwhelmingly opposes the water
system. That also shows that the evidence in question is not In dialog-based Method of Argument questions, first read
“impossible to disconfirm,” as (D) would have it. the part of the stimulus for which you are asked to find the
method, especially if the two speakers’ remarks are about the
23. (A) same length.
The correct answer to a Method of Argument question will be We must describe Quincy’s method of argument, and because
a 1:1 matchup between the stimulus and the answer choice. Quincy says quite a lot, the odds are in our favor that we can
predict the answer by reading Quincy’s remarks alone. If,
This question, which deals with a proposed Factory Safety
by contrast, Quincy uttered only two or three lines, it would
Act, includes the most obvious Formal Logic statement of the
definitely be necessary to read both arguments. Quincy
entire section, with an “only-if” in the first sentence. We know
questions what is probably Phoebe’s conclusion, and, getting
from translating this statement that if a company operates
more specific, says that a theory (probably central to Phoebe’s
an auto factory, it must register that facility as a “class B”
argument) is “speculative,” or in other words, not proven. So,
factory. The second statement tells us that class B factories
we can characterize his reasoning in general terms: he objects
to Phoebe on the ground that she uses a dubious theory as to kill insects. So, her point is the same as his, but she cites
her central support. Choice (C) says Quincy’s attack is about alternative evidence for it, and that’s (E).
part of Phoebe’s argument being “unsubstantiated,” or in Jennifer can’t be supporting (A) or explaining (C) Peter’s claims
other words, unsupported, and is thus correct. because each person’s evidence focuses on a different factor.
(A) is a 180—Quincy criticizes Phoebe for drawing a causal And as such, she can’t be supplying a missing premise (B)
conclusion from correlation evidence. This answer is more either. Meanwhile, “indeed/in fact” tells us that Jennifer is
tempting for students who read both passages rather than in the business of reinforcing Peter’s recommendation, not
focusing only on Quincy. Expect that in a dialog-based Method undermining it (E).
of Argument question one of the wrong answer choices will
focus on the wrong person’s reasoning. 28. (A)
(B) is incorrect because Quincy doesn’t appear to challenge “Counters . . . by” tells you to assess the approach or method.
the accuracy of any data (which would be evidence)—he only Why, according to the conservationist, is the wildlife refuge
challenges the theory on which Phoebe’s argument is based. “therefore” no safety risk? Only 20 damaged planes in
Quincy does not offer any kind of alternative explanation for 10 years, and no passenger injuries. But the pilot sheds new
anything, so (D) is incorrect. light on those stats: Most of the 20 are extremely recent, and
(E) is wrong because Quincy doesn’t call anything “irrelevant.” the bird population is skyrocketing. In other words, the context
of the conservationists’ facts suggests that the refuge is not a
26. (B) place of safety, but a place of tragedy just waiting to happen.
It all amounts to identifying a misleading use of statistics,
When the question stem asks about one of two speakers,
(A). The most tempting wrong choice is (E), but note that the
define the important speaker’s objection before scanning the
pilot accepts the accuracy of the statistics. He merely suggests
choices.
that they have a very different meaning.
Reading Campisi first, we see that Campisi thinks an
(B) accuses the pilot of questioning the conservationist’s
“inference” Yang makes is unconvincing, and offers what
motives, but correcting an opponent’s facts isn’t the same
appears to be an alternative. An inference is something that
as impugning her motives. The greater danger that the pilot
is unstated. Yang says people knew yeast was a leaven at
predicts isn’t “inevitable” because of time passing, (C),
least as early as 1200 B.C. because they used leavens at that
but rather the inevitable result of the very facts that the
time. Yang’s assumption, then, is that it was yeast that they
conservationist used to make her safety claim. The pilot
were using, an assumption that Campisi challenges, which
appeals to hard facts, not to morality, (D).
leads us to (B).
(A) Campisi’s “alternative” isn’t a “set of evidence” nor 29. (A)
does it “support Yang’s conclusion”—it works against Yang’s
The author’s sheer purpose is to counter the claim made
conclusion.
in the first sentence by providing another reason—or “an
(C) Campisi does consider the reason given and argues alternative explanation,” in (A)’s words—why physicians
specifically that it fails to prove Yang’s point. don’t prescribe herbs.
(D) Campisi never suggests any contrary conclusion, and The data (if any) used by the people mentioned in sentence 1
certainly does not say anything about a contrary conclusion goes unmentioned, let alone attacked (B). The author doesn’t
being more strongly supported by Yang’s evidence. see an inconsistency (C) in the claim that physicians mistrust
(E) Campisi does not challenge the truth of any information herbs; she just sees the story from a different angle. Many
Yang gives. Campisi challenges the (unstated) assumption other explanations for the physician/herb situation (D) go
Yang makes. unmentioned here, and (E) would be closer if it said that the
author rejects one theory (if we can call it that) by proposing
27. (E) another one. But that would turn (E) into (A), after all.
Peter’s argument is one for underwatering, which he implies
makes leaves less desirable to insect predators than the less- 30. (B)
tough leaves produced by abundant watering. “Indeed, in A short response most likely focuses on a narrow segment of
fact” signals Jennifer’s agreement with Peter’s conclusion that the first argument; zero in on that narrow focus to paraphrase
farmers should not overwater. But her evidence comes from a the answer.
different but related place: She approvingly cites the greater The mall manager is concerned by the large groups of
likelihood that underwatered plants will develop poisons teenagers congregating in his mall, and argues that their
presence causes adults to spend less time shopping in the
mall’s stores. He uses this information to conclude that that the anthropologist was ignorant of the possibility of
merchants in the mall should discourage teenagers from drawing an analogy between animals and machines.
congregating near their stores, giving as a reason the fact that (E) Any sort of misinterpretation of terms is outside the scope
the mall’s goal is to prevent a loss in sales. of both arguments.
The merchant (perhaps one of those being told to kick the
teenagers out?) counters that the teenagers themselves 32. (D)
account for a significant percentage of the total sales in The correct choice for a Method of Argument question must
the stores. This response is short and to the point, so we match the stimulus point for point.
can quickly figure out which part of the mall manager’s
James is jazzed about a new technique for extracting rhodium
argument it responds to. Highlighting the amount of money
from nuclear waste. Rhodium is used to make catalytic
spent by teenagers casts doubt on the manager’s reasoning
converters, and catalytic converters in turn make cars cleaner.
by suggesting that kicking out the teenagers would not
And so, James argues, nuclear power is helping to make the
further the manager’s goal of avoiding a loss in sales,
environment cleaner. How does Marta respond? That’s all fine
because getting rid of the teenagers would mean losing their
and good, but the technique is actually still in the preliminary
business. (B) is a perfect paraphrase of this relationship as
stages and hasn’t been carried out yet. Marta agrees with all
described by the merchant.
James’ evidence, but not his conclusion.
(A) Everything the manager claims as support for his
(A) Marta has no argument with the evidence James mentions,
recommendation could still be true. The merchant adds
but with his conclusion. Eliminate.
another consideration that weakens his conclusion.
(B) Marta never mentions advocates of nuclear power, so she
(C) The merchant’s argument implicitly agrees with the mall’s
can hardly be questioning their credibility. She only questions
goal, suggesting that the manager’s recommendation isn’t the
James. Eliminate.
best way to achieve it.
(C) accuses James of using circular logic. No matter how
(D) The merchant’s response suggests that the manager’s
deeply flawed you consider his argument, the evidence and
recommendation isn’t sound at all. (D) is a 180.
conclusion of his argument are distinct, and by definition
(E) The merchant makes no new recommendation. cannot be circular. Eliminate.
(D) is exactly what Marta does. She points out that James
31. (C)
never mentioned the fact that the technique is still on the
The anthropologist details an experiment he was a part of drawing board.
and the conclusions that experiment arrived at. In short,
(E), like (A), suggests that Marta disagrees with James’s
the experiment involved mapping out the hierarchy for a
evidence. She only points out something he failed to mention
troupe of monkeys, then using that hierarchy to predict the
and does not question the accuracy of what he did mention.
behavior of the monkeys. But in making his predictions, the
anthropologist informs us, he made one key assumption:
33. (A)
that every monkey knew the hierarchy in the same way that
the anthropologist did. He concludes that this assumption is This question asks you to determine the way Anika’s response
correct from the fact that his predictions were correct. flows, and her statement “I disagree with your prediction” is
a great clue. Inez’s prediction is that having antiques
But the primatologist begs to differ, and does so by way
professionally appraised would make customers more willing
of analogy. His analogy leads to the conclusion that bank
to buy the antiques, because it would remove the possibility of
cash machines really have desires, simply from the fact that
getting suckered into purchasing fake items. Anika disagreeing
we can predict their behavior by assuming that they have
with the prediction is a pretty blatant sign that she believes
desires. Such a blatantly false conclusion, reached in the
the opposite—this plan would not make customers more
same manner as the anthropologist’s conclusion, casts doubt
willing to buy the antiques. And the rest of her statement
on the anthropologist’s argument. (C) reflects this method of
bears this out: the customers are already experts, so hiring
argument-by-analogy.
a professional would only raise costs, causing people to buy
(A) The facts that the primatologist cites could still be true if fewer antiques. Pretty straightforward: Anika feels that Inez’s
the anthropologist’s assumption was really correct. plan would have the opposite of the anticipated effect, which
(B) The primatologist doesn’t deny the anthropologist’s is (A) precisely.
conclusion as a premise of his argument, but as the (B) is a half-right, half wrong choice. While Anika feels
conclusion of his argument. the plan shouldn’t be adopted, she doesn’t see anything
(D) The analogy that the primatologist uses doesn’t undermine effective about it.
the anthropologist’s expertise in any way, nor does it suggest
(C) Alternative plan? Not there. particular guy already knew Homer by heart, why on earth
(D) Anika never attacks the appraisers’ qualifications—just the would he bother to write it down? Anyway, M’s idea is
added cost that hiring them would entail. ridiculous and self-contradictory, P implies—or “absurd,” as
(D) has it.
(E) Anika doesn’t offer any counterexamples, and never claims
that Inez’s plan is too general. In fact, Inez’s plan is quite (A) “Literary value” has nothing to do with either M’s
detailed—Anika just doesn’t think it’ll work. hypothesis or P’s response. This dispute is about how an
alphabet came into being.
34. (C) (B) P does disagree with M’s idea, but he attempts to refute it
Next, we’re faced with another disagreeing pair, Sarah and too—by citing as evidence its inherent absurdity. Although P’s
Paul. And Paul is up front in his opposition as he flatly argument is flawed, he does indeed provide rebuttal evidence.
states “I disagree.” Paul disagrees with Sarah’s opinion that (C) P never even mentions M’s presumption of a Phoenician
programs that require students to volunteer don’t succeed basis for the Greek alphabet, so we can’t say that that is the
in producing people who like to volunteer because those “laughable” part, in P’s eyes.
students are forced to participate. Au contraire, says Paul: (E) Strictly speaking, M is positing a hypothesis rather than
Some students enjoy their community service so much that constructing an argument with evidence and conclusion; in
they eventually decide to volunteer on their own. In other other words, technically M isn’t citing any evidence at all. Be
words, he brings up a situation which breaks down Sarah’s that as it may, P doesn’t reinterpret anything M says; he just
assumption that all volunteering is necessarily forced tries to show that M’s hypothesis makes no sense.
volunteering. Choice (C) gets the point.
If you weren’t able to pre-phrase an answer, at least you may 36. (A)
have been able to recognize what Paul is not doing, and thus The issue at hand, we learn from sentence 1, is: What has
eliminate all the losers: been the effect of a region’s high unemployment on the
(A) Paul doesn’t accuse Sarah of employing circular logic; that employed people there? Rumor seems to have it that even the
is, assuming what she sets out to prove. He believes that the working folks have cut back on their spending, as if they too
conclusion she derives from her evidence is faulty, not that were in financial trouble. But the author rebuts that rumor:
she hasn’t provided any distinct evidence at all. “Clearly,” he concludes, workers are spending as usual. What
(B) Paul doesn’t challenge Sarah’s conception of volunteering, makes him think so is that workers’ savings accounts are
but rather her notion of the results of forcing students to about normal. He seems to think that if the working folks were
volunteer. If (B) were meant to be correct, Paul would probably cutting back on their spending, they’d be putting more money
have mentioned specific examples of activities that Sarah away in savings. And since they’re not saving more (see the
didn’t consider to be volunteer activities that he would contrapositive at work here?), they must not be cutting back on
include, but nothing of the sort appears in his argument. spending.
(D) Sarah’s motivations are far from Paul’s mind in making his This Method of Argument question wants us to translate that
argument. For (D) to be correct, he would have had to attack analysis into abstract terms. One way you might approach
Sarah’s motivations directly, something along the lines of: it is like so: If a certain thing were happening (e.g., less
“You only want to see the volunteer program cut so that you worker spending), we’d expect to see a certain effect (e.g.,
have more money budgeted for the school orchestra!” bigger savings accounts). Since we haven’t seen that effect,
the original thing must not be happening. This is in so many
(E) Paul discusses an outcome, not a policy, that Sarah fails
words what (A) gives us—since an effect isn’t visible, its cause
to consider.
cannot have occurred.
35. (D) (B) Contrary to (B), the author’s conclusion is not whether
behavior X caused observed-occurrence Y, but whether or not
If, when two people are talking in a stimulus, you’re used to
a particular behavior—spending by employed people—is or is
reading the second person first, simply seeing that P calls M’s
not diminished.
hypothesis laughable might have caused you to stop right
there and check out what M’s hypothesis is. The issue is, who (C) is a huge distortion of the stimulus. The author makes no
invented the Greek alphabet? M’s idea is that it was someone judgments as to whether spending behavior in the region is
who knew how to write Phoenician and wanted to record and good or bad, well-informed or ignorant.
preserve Homer’s cool (but orally transmitted in Greek) poetry. (D) The author doesn’t conclude that “one of two alternative
This idea is laughable, according to P, because the inventor developments occurred.” The author definitely believes
of the Greek alphabet by definition must have been the that actual spending is undiminished and so the author’s
only person who could read Homer in Greek; and since that conclusion is much more specific than (D) would have it.
(E) While the author does argue that no change in spending is Darla’s claim is merely meant to undermine the way Charles’
actually occurring, it’s not because of “ambiguous evidence.” support leads to his conclusion.
It’s because a specific predictable effect of less spending— (D) Au contraire, Darla expresses skepticism about the
greater savings—is explicitly not present...and that brings us conclusion itself.
back to (A).
(E) What absurd conclusion?
37. (E)
39. (D)
Since we know from the stem that we’re looking for the
The issue is mandatory sentencing, which the politician
method of argument, we can focus on how the astronomer
esteems as a positive vehicle for justice, since (to him) it
makes her argument. The keyword “therefore” indicates the
makes justice less subject to whether a judge happens to
conclusion. It is possible to have well-lit streets and relatively
be strict or lenient. The advocate wants to see it repealed
dark skies. How does the author back up this claim? She
because of its effects on juries, who (she argues) may acquit
notes that many people believe that interference from light
not because they’re convinced the accused is not guilty, but
is unfortunate but necessary, since people need bright lights
because they feel the mandatory sentence would be too
for safe streets. “Here in Sandsville, however,” indicates that
harsh. So, the advocate’s response, as (D) says, examines
the example of Sandsville will be used to counter that claim.
the consequences of the policy in an effort to show that
And it does. Sandville’s experience demonstrates that it is
mandatory sentencing doesn’t ensure, but rather perverts,
possible to have it both ways, so the astronomer does offer a
justice.
counterexample to a general claim, (E).
(A) would only be accurate if the advocate were accusing the
(A) What scientific authority? No one’s expertise is at issue.
politician of circular reasoning. But her point deals with the
The author is an astronomer, but she doesn’t rely on her
ramifications of the proposal, not the politician’s repeating
expertise to make her argument.
himself.
(B) The astronomer doesn’t question whether certain
(B) The advocate doesn’t deny the relevance of the politician’s
observatories have become useless due to interference, she
evidence; instead, she brings up evidence she feels he has
just argues that this need not always be the case.
overlooked.
(C) No explanation of a scientific phenomenon is at issue.
(C) distorts the advocate’s first sentence, which is not meant
Everyone understands why street lights interfere with
to assert (as (C) has it) that leniency varies depending on the
observatories.
nature of decisions, but simply that “leniency” is an irrelevant
(D) What distinction? concept when mandatory sentencing is imposed.
(E)’s charge of exaggeration seems unsupported. If anything,
38. (C)
it’s the advocate who is arguing that a problem is severe.
Charles indicates that during recessions a greater percentage
of people are out of work. The keyword “thus” signals the 40. (B)
conclusion: During a recession air pollution caused by cars
The issue here concerns the method by which female bats
decreases. Why? The keyword “since” indicates supporting
reunite with their “pups” amongst millions of bats in Bracken
evidence follows: Fewer people are commuting, which means
Cave. The only possible way to find one another is through bat
fewer cars and therefore less pollution. Darla then challenges
calls (by way of the Bat Phone?), but the pups are clueless—
the conclusion, but she doesn’t challenge Charles’ evidence.
they can’t tell mommy from a hole in the wall. But since
Instead, she brings up a competing consideration: If fewer
the moms do hook up swiftly with their pups upon return,
people have jobs, then fewer people have new cars, which
the author concludes that the mother bat must be able to
means more old cars are on the road. Old cars pollute more,
recognize the call of her pup. What, in general terms, is the
which could mean more pollution. Generally speaking, Darla
argument trying to do? It asserts a conclusion regarding a
undermines Charles’ argument by introducing an additional
phenomenon by ruling out the only other possible explanation
consideration, (C).
for the phenomenon. In other words, the mother bat must be
(A) Darla doesn’t contest Charles’ evidence; she just claims able to recognize its pup’s call (establishing the validity of one
that it doesn’t justify his conclusion. explanation) because bat calls are the only way for bats to
(B) is tricky: It implies that if air pollution from cars decreases find each other, and the pup can’t recognize the mother’s call
during a recession (Charles’ conclusion), then it’s impossible (excluding an alternative explanation)—choice (B).
that fewer people can afford new cars during a recession (A) The author doesn’t argue from part to whole, from small
(Darla’s claim). But that need not be true—there’s no implicit to large, based on a representative sample, as (A) asserts. It’s
contradiction between Darla’s claim and Charles’ conclusion. simply not there.
(C) has it backwards: The stimulus argues from the existence it’s fairly simple and certainly not used as a means of
of a phenomenon (mother bats reuniting with their pups) argumentation.
to a conclusion regarding the nature of the mechanism (bat (B) , (D) No experimental results were used in the argument;
calls). Maybe you were able to eliminate this choice simply the sole support for the argument is the statement by a single
by recognizing that the phenomenon (reuniting) is not a authority.
hypothesis, but a given.
(E) is way off the mark. The conclusion has nothing to do
(D) What are the two groups? There’s only one group, bats, with accepting a theory about “all regions of space” based
broken down into two kinds (mother and pup) that expressly on a generalization. In fact, the author rejects a theory as
don’t share the same ability. incoherent based solely on one “expert’s” opinion.
(E) Even if we allow “reuniting” to be deemed a “general rule,”
there’s still no particular case to which this “rule” is applied. 43. (E)
Lydia’s conclusion is that squirrels chew the bark to get to
41. (D) the sugar. She supports this by stating that they are either
Does Mary come right out and tell Tom that it’s a terrible idea? after sugar or water, and it doesn’t make any sense for them
No. But she does introduce a new consideration that certainly to use sap to get water, so they must be after sugar. In other
puts a damper on his proposal. Tom believes that high school words, she presents two possibilities, and rejects one of the
students should work part-time jobs to help prepare them possibilities, leaving only the one that she endorses. This
for the work world, but what if these job opportunities simply method is summed up nicely in (E): she rejects “a possible
aren’t available? He assumes that the opportunity for part-time alternative explanation for an observed phenomenon.”
work exists for high school students, but Mary strongly implies Although she dismisses the water theory, Lydia never
that this may not be the case. (D) describes her response dismisses any actual data (A). There is also nothing in the
perfectly: she presents a consideration (the lack of part-time passage which could be described as a general rule (B) or a
job opportunities for high schoolers) that undercuts Tom’s larger pattern (C), since she is dealing only with the specific
basic assumption (that high-schoolers can get part-time work). behavior of eating sap. Finally, (D) is off base, since nothing in
(A) No, Mary never says “here’s a really bad thing that will the passage could be described as an analogy (a comparison
happen if we require high school students to work part-time of something unfamiliar to something more familiar).
jobs.”
(B) implies that Mary is arguing “why bother, Tom—this trend 44. (B)
(lack of vocational skills among graduated high schoolers, if The author bolsters her belief as to the true cause + effect by
we can call that a trend) is unavoidable.” Mary does no such citing scientific studies (the “additional evidence” of which
thing. (B) speaks), studies whose results question the sentence 2
(C) All Mary does is present a new consideration that hammers evidence that people think supports the “widespread belief”
away at Tom’s implicit assumption. Nothing in her response of sentence 1. Each phrase of (B)’s has a parallel in the
contradicts—that is, is inconsistent with—any explicit part of argument.
Tom’s argument. All five choices look good without a prephrased idea of the
(E) Mary’s a great devil’s advocate, knocking down Tom’s right one. The author doesn’t “question the accuracy” of
proposal on the grounds that it may be impossible to carry the chocolate/acne correlation; rather, she reinterprets that
out. But savior, she isn’t: nowhere does she offer her own connection, so (A) distorts the logic. Since the author accepts
solution to the problem Tom cites. the chocolate/acne correlation that is part of “everyday
experience,” she is hardly “dismissing it out of hand” (C), nor
42. (C) is she relying on the mere authority of science over common
sense. The chocolate/acne correlation is consistent with
This argument begins by describing an eighteenth-century
science’s view of the role of stress, so (D) is a 180. And (E)
theory, and continues by asserting that a “well-respected
seems to think that the author is concerned with the time
physicist” has deemed the theory to be incoherent. The author
sequence of cause + effect—i.e., which “precedes” the other?
then rejects the theory on the grounds that it’s incoherent.
— when that has no role in her logic whatsoever.
The only support for rejecting the theory is the statement by
the physicist. (C), therefore, best describes the method of
45. (A)
argumentation.
The objection to the definition is that in the case of
(A) Although there is some technical terminology in the
clairvoyance, meeting the sufficient condition of “true belief
argument (motion is absolute, object’s position over time),
formed by reliable process” would still not make people
accept clairvoyant statements as “knowledge”—hence, to the 47. (D)
objectors, the conditions aren’t sufficient and the definition Keywords help you navigate your way through Dillworth’s
of knowledge is faulty. The author’s rebuttal wisely points unusually lengthy diatribe. Situation: People are deciding
out that the objection shifts the scope: Clairvoyance isn’t against kids for two reasons: a lot of sacrifice is required,
accepted as reliable (if it were, we would accept its claims as and the kids end up ungrateful, grumble grumble. Contrast
knowledge), so the counter objection isn’t on the mark. (A) Keyword “However”: He brings up another consideration
explains that. that, in his mind, outweighs the first: the fact that kids are
(B) is a 180 because the author asserts that the unreliability the best way of passing on one’s values. Conclusion Keyword
of clairvoyance makes it impossible to yield knowledge. (C) “Therefore”: Dillworth’s point is that those who hold values
is a 180, too, because she defends rather than rebuts that should separate themselves from the pack mentioned in
unreliability. Since the question stem itself reveals that the sentence 1, and have kids despite all the sacrifice and
author is defending the definition, she’d hardly set out to ingratitude. (D) puts all of that in abstract language. The
reframe that definition as (D) suggests. And (E) falls short “category” is people with deep values, the “given course of
in two ways: It ignores the fact that the author says that she action” is having kids, and the “reasons cited against [it]” are
could concede clairvoyance as yielding knowledge (if it were the sacrifices and ingratitude.
seen as reliable); and it simply doesn’t speak to the defense of (A) is tempting to those who read choices too quickly,
the definition against the objection, as the question demands because it sounds superficially plausible. But Dillworth
(and as (A) does). doesn’t deny that the sacrifices required to raise kids, and
the resulting ingratitude, are real “drawbacks,” as (A) would
46. (E) have it. Instead, he takes them off the table, saying that
The administrator’s first word, “But,” hints that she is going those drawbacks “have no bearing on” what should be the
to take issue with the auditor’s point of view, and so she compelling reason for action, namely the desire to pass on
does. But how? She believes that the money is in fact working one’s values.
to alleviate patient suffering, through the development of (B) The choice Dillworth rejects—not having kids—he rejects
diagnostic tools that will minimize the suffering of patients only for those who hold deep values. And he doesn’t pass
who’d feel much worse if their disorder were discovered late moral judgment on everyone else who opts against child
rather than early. rearing.
The choices here confound prephrasing a bit, because they (C) Dillworth says nothing about prohibited actions.
indulge in a lot of confusing abstract verbiage. But wrestling
(E) Just a lot of jargon—there’s no “specific-to-general”
the language down to the ground should lead you to (E). The
reasoning here. If anything, Dillworth argues that a course of
“key phrase” that the administrator “reinterprets” in order to
action appropriate under one set of conditions would not be
undermine the auditor’s assumption is as follows: “instead
appropriate under another.
of letting that money flow directly to [the] patients.” As we’ve
seen, the auditor assumes that money devoted to research is
48. (B)
not, at the same time, “directly flowing” to the relief of patient
suffering. And that’s the belief that the administrator attacks. Reading and analyzing a question stem carefully can render a
question much easier than it might at first appear.
(A) A direct assault on a key assumption hardly amounts to “a
minor qualification.” The stem asks for “a” technique used by the philosopher, not
“the” technique; and as soon as we read “An unnatural action
(B) The auditor clearly separates what in his mind ought to be
is either...” we can see that he has employed technique (B):
the use of the money, and how the money is being used, and
yes, he has defined that key term, namely “unnatural action.”
the administrator never challenges the auditor on that point.
No other understanding of the dense stimulus is required,
(C) goes against what the administrator is saying. The solely because we noticed the “a” rather than “the.”
administrator points out an alternative way to satisfy the
(A) The only concept the author might be accused of
family’s wishes, but (C) implies that the administrator believes
undermining is the concept of “unnatural action.” But he does
that there is only one solution to the problem. The auditor is
so not by alleging that it violates nature’s laws, but by arguing
the only one that might agree with the notion in (C).
that no such action can exist, that neither definition of the
(D) The administrator concedes no violation of the Rodriguez concept makes any sense.
family’s stipulations, so she would in no sense be evaluating
(C) No statistical findings are offered. The phrase “statistical
the severity of same or examining an underlying motive.
anomaly” is simply a fancy way of saying “it doesn’t happen
very often”—the second possible definition of “unnatural
action” offered by the author.
(D) The philosopher surely does set out to undermine the claim temperature and shows that they’re both effects of some other
he cites in the first sentence, but he does so not by pointing out cause. That’s clearly not true; no ultimate cause is mentioned.
an internal contradiction, but by denying that there can be such (D) The meteorologist never criticizes the statistician’s
a thing as an unnatural action in the first place. conclusion because it can’t be tested; rather, she criticizes it
(E) No empirical evidence is provided. Moreover, the because it contradicts an accepted rule.
philosopher systematically rejects each possible definition of (E) The meteorologist doesn’t come close to saying “you
“unnatural action” without ever supporting one over the other. purposely left out evidence that would damage your
argument.” But this is what she would have to have said if (E)
49. (A) were meant to be correct.
Stephen is playing good GMAT test-taker in his response.
Zachary’s argument concludes (“Therefore”) that to return the 51. (A)
Sistine Chapel to what Michelangelo intended, it’s necessary Now we’re asked for the cold sufferer’s method of argument,
to strip away “everything except” the original fresco work. so let’s check the choices against the argument until we find
The evidence for this is that later painters made additions. a match. Choice (A) has a few different parts, so let’s take it a
Stephen identifies the gap between evidence and conclusion: bit at a time. Does the cold sufferer find a claim to be false?
What if Michelangelo himself had made additions after Sure; our grumpy cold sufferer is skeptical of the claim that
his original work, in order to fulfill his intentions? (And it the herbal mixture is an effective cold remedy, and in fact
sounds as if he may well have done so.) If so, stripping away proceeds to pronounce this claim false. So far so good. But
“everything except” the original work would not achieve does the rest hold up? Yes: The cold sufferer denounces the
Michelangelo’s intentions at all. Stephen is doing exactly what claim because he feels that, if the claim were true, there would
you should be doing on a large portion of Logical Reasoning be consequences (almost everyone with a cold would be using
questions: identifying the assumption (A). it) that in fact are false (many people with colds don’t use it).
(B) suggests that Stephen challenges Zachary’s terminology, All elements match up, so (A) fits the bill of the cold sufferer’s
which simply isn’t the case; he accepts Zachary’s terms with method of argument.
no demurral. While Stephen is throwing in a possibility that (B) The cold sufferer denounces the claim; he doesn’t
endangers Zachary’s conclusion, he is not reaching one of his accept it.
own (C). Nor does Stephen attack any of the evidence Zachary
(C) No, here the author specifically constructs conditions that
presents (D) or suggest that the conclusion doesn’t follow
are met in order to establish the falsity of a claim.
logically from this evidence (E). Instead, Stephen adds new
evidence that points out and questions, without denying or (D) Where’s the generalization? There isn’t one. Every part
demonstrating, the assumption in Zachary’s argument. of the argument is related specifically to the cold medicine
in question.
50. (A) (E) The author certainly argues that a measure claimed to be
The statistician argues from correlation to causation: the effective would not be effective, but he never argues that this
Sun’s luminosity correlates with Earth’s land temperatures, measure (the mixture) would make matters worse—that “it
therefore the Sun’s luminosity determines Earth’s land would actually make achieving the effect more difficult.”
temperatures. The meteorologist doesn’t attack this reasoning,
as you might expect, but instead simply declares that the 52. (D)
conclusion is impossible because it’s a recognized rule among Since W draws no conclusion, the facts she cites make sense
meteorologists that in a complex system (like the Earth’s only in the context of S’s argument that we shouldn’t try to halt
climate) no significant aspect (like land temperature) can be future global warming because scientists disagree all over the
controlled by a single variable (like the Sun’s luminosity). As place; the evidence isn’t firm; global warming may not even
(A) says, the meteorologist invokes a relevant generalization be real. What W does is suggest that scientists don’t bother
(The above-mentioned rule) to draw a conclusion about this to argue when facts are established, and that scientists do
particular problem. accept one fact: global warming is happening, albeit within
(B) The meteorologist doesn’t produce a counterexample an uncertain 3°C range. W’s facts are the “similarities” among
(which would be something like a case where a change in the scientists to which (D) refers.
Sun’s luminosity isn’t accompanied by a change in the Earth’s (A) W and S are equally aware of the disagreements among
temperature) but relies on a general rule. scientists; it’s just that W denies those disagreements’
(C) essentially says that the meteorologist analyzes the relevance to the global warming debate.
correlation between the Sun’s luminosity and Earth’s (B) Since W doesn’t bring out her explicit point of view, we
can’t be sure that W accepts S’s conclusion—indeed, their
profound disagreement over the global warming trend implies
that their conclusions would be very different.
(C) Inferably, W’s scientists are the same as S’s. They just look
at these authorities from different perspectives.
(E) Another accusation of circular reasoning, but this one
is balderdash. First of all, W doesn’t explicitly offer a
conclusion at all; also, her evidence is quite concrete with
no hint of “circularity.”
53. (D)
The stimulus begins with the psychologists’ claim that
people are more susceptible to psychological problems in
the winter than in the summer (seasonal affective disorder).
The only support cited for this claim comes from the survey.
The psychologists are assuming that the self-reported results
of the survey are accurate. The author disagrees with this
assumption and uses the word “however” to signal that
she disputes the validity of the survey results because she
does not believe that self-reported results about the past
are accurate. The author, “therefore,” does not believe that
the survey results support the existence of the disorder. The
author disputes the psychologists’ conclusion by questioning
their central assumption that the results of such a survey can
be accurate—choice (D).
(A) The author believes the survey results are suspect, but
she never ventures her own explanation of the change in
susceptibility to psychological problems across seasons. In
fact, in the end she denies that the disorder exists, so why
would she offer an explanation for it?
(B) is way too broad: our author questions the existence of
such a disorder because she rejects the methodology of this
particular survey. That’s not the same as her questioning
whether any such variation, under any circumstances, could
be labeled a disorder—that would be a few steps beyond this
specific argument. In other words, the issue to the author isn’t
whether or not such variation across seasons in psychological
states constitutes a disorder, as (B) would have it; she out-
and-out denies the existence of the variation altogether
because she doesn’t trust the survey results.
(C) Although the author does not believe that the survey
supports the psychologists’ conclusion that the disorder
exists, she never questions whether or not the survey
sample is representative—she instead questions the survey
respondents’ ability to accurately recall how they felt.
(E), like (A), implies that the author believes in the existence
of seasonal affective disorders, but we know that the author
really feels that the survey evidence doesn’t justify the claim
that such a condition exists.